[obm-l] Re: [obm-l] ensino de matemática

2018-07-11 Por tôpico Carlos Nehab
Bem, Claudio,

A gente se conhece por essas bandas há tempos.

Subscrevo suas observações e, motivado por cafezinho, chopp, e/ou  outras
cabeças pensantes, até ousaria  complementá-las. Rsrsrs.

Sim, tenho MUITO interesse em pensarmos juntos.

Grande abraço
Nehab



Em Qua, 11 de jul de 2018 12:38, Claudio Buffara 
escreveu:

> Prezados colegas da lista:
>
> Entendo que o tema pode ser off-topic pois não trata especificamente de
> problemas olímpicos, mas aqui vai de qualquer forma...
>
> Algum de vocês se interessa pelo ensino de matemática (escolar ou
> universitário)?
>
> Pergunto porque há anos tenho pensado na melhor forma de ensinar
> matemática (principalmente em termos de composição do currículo e de
> apresentação dos tópicos nos livros didáticos), estou convencido de que não
> estamos fazendo certo, nem na escola e nem na universidade, e gostaria de
> ter gente interessada pra debater idéias e, quem sabe, elaborar algum
> projeto mais concreto.
>
> Em linhas gerais, discordo da ordem em que os assuntos são abordados, na
> maioria dos livros.
> O foco é muito mais na ordem lógica (seguindo o rigor do método
> axiomático, mesmo em livros pra ensino médio) sem nenhuma preocupação:
> - com a motivação para os resultados que são apresentados (e, nos ensinos
> fundamental e médio, quase nunca demonstrados);
> - com tornar estes resultados intuitivos para o estudante.
>
> Também acho que certos assuntos deveriam ser incluídos e outros excluídos
> do currículo, mas este, pra mim, é um problema menor. Pois, qualquer que
> seja o tópico, se for bem ensinado e incentivar o aluno a pensar, já tá
> valendo.
>
> A meu ver, seria ideal se cada tópico do currículo de matemática fosse
> apresentado seguindo a sequência:
> identificação de padrões ("patterns") ==> formulação de conjecturas ==>
> demonstração destas conjecturas.
> Pois esta é a maneira como a matemática é criada.
> Mas acho que muito poucos professores estão capacitados pra ensinar
> matemática deste jeito.
>
> Em particular, no Ensino Médio, a ênfase nos últimos anos tem sido na tal
> contextualização, que pode ser vista em todo o seu esplendor nas provas do
> Enem.
> O resultado disso me parece ser um retrocesso na formação matemática dos
> alunos e também a disseminação da mentalidade de que a única matemática que
> deve ser estudada é aquela que é usada no dia-a-dia dos cidadãos comuns.
>
> E, na universidade, a coisa não é muito melhor, mesmo num assunto que só é
> visto na graduação em matemática. a análise real.
> Vejam só:
> Os livros tratam da topologia da reta antes de conceitos tais como
> compacidade e conexidade se mostrarem realmente necessários (o que, de
> fato, só ocorre em dimensão > 1; na reta, quase tudo pode ser demonstrado
> com base em sequências e no método da bisseção, que são coisas bastante
> intuitivas, mas que quase nunca são usadas).
>
> Limites e continuidade podem ser introduzidos também com base em
> sequências, interpretando-se os epsilons como margens de erro em
> aproximação.
>
> Aliás, a noção de que análise nada mais é do que uma teoria de
> aproximações quase nunca é mencionada.
> Por exemplo, foi só estudando a análise do R^n é que eu me dei conta de
> que a derivada é uma aproximação de uma função arbitrária por uma função
> afim.
> Antes disso, eu só sabia que "derivada = inclinação da reta tangente".
>
> Os livros também mencionam critérios de convergência de séries (Dirichlet,
> Abel, etc.) que vêm do nada (pois foram inventados para o estudo de séries
> de Fourier, que estes liros não abordam).
>
> E o principal resultado sobre convergência de séries de potências decorre
> quase trivialmente do estudo das PGs infinitas (assunto de Ensino Médio).
> Mas qual livro deixa isso explícito?
>
> E, pra terminar, poucos têm uma figura para ilustrar o teorema fundamental
> do cálculo que, com uma figura bem feita, fica bem intuitivo. No entanto, a
> análise na reta em geral é apresentada com um caráter aritmético/algébrico,
> mas quase nunca geométrico.
>
> Obrigado pela atenção.
>
> []s,
> Claudio.
>
> --
> Esta mensagem foi verificada pelo sistema de antivírus e
> acredita-se estar livre de perigo.

-- 
Esta mensagem foi verificada pelo sistema de antiv�rus e
 acredita-se estar livre de perigo.



Re: [obm-l] Perguntas pro Claudio Buffara

2018-04-12 Por tôpico Carlos Nehab
Prezada,

Como alguns amigos já responderam alguns aspectos de suas perguntas, vou
responder ao mais simples.

Sim, Produtos Notáveis são mal ensinados e, em geral, burramente descritos
nos livros. Em geral, um amontoado de identidades chatíssimas sem nenhuma
utilidade para as crianças e adolescentes, assim como dezenas de outros
assuntos de Matemática. Por isso, basicamente, o completo desinteresse da
maioria dos alunos jovens nessa disciplina.

Para não me alongar, ficam algumas perguntas:
- como vc calcularia, mentalmente, 70 x 13, por exemplo?
- como voce calcularia 13 x 13, mentalmente?
- como vc calcularia 19 x 21, mentalmente?
- como vc calcularia 27 x 27, mentalmente?

Se vc respondeu que é usando distributividade e alguns produtos notáveis,
está num bom caminho motivador para seus alunos acharem sua aula
minimamente interessante.

Talvez, mais tarde, eles venham a amar essa lista que, em geral, é um
divertido quebra-cabeça e povoada por criaturas aparentemente estranhas.
Como eu, que dela participo há décadas. Rsrsrs.

PS: Até prova em contrário, embora haja algumas controvérsias, sou um bom
sujeito e um sujeito aparentemente normal. Juro.

Abraço.
Nehab

Em Ter, 10 de abr de 2018 13:17, Marcela Costa 
escreveu:

> Caros participantes da lista obm-l.
>
> Tenho seguido esta lista lendo as mensagens de fora há algum tempo e
> fiquei cismada com duas mensagens que o participante Claudio Buffara enviou
> em 23 de março (
> https://www.mail-archive.com/obm-l@mat.puc-rio.br/msg55232.html ) e 25 de
> março ( https://www.mail-archive.com/obm-l@mat.puc-rio.br/msg55196.html),
> a respeito do ensino de matemática e decidi participar.
>
> Dessa forma, tenho as seguintes perguntas pra ele:
>
> 1) O Sr. diz que produtos notáveis e fatorações são "notoriamente mal
> ensinados". O Sr. tem alguma sugestão de como ensinar melhor estes tópicos?
>
> 2) O Sr. não acha um pouco arrogante fazer uma afirmação como esta, já que
> o Sr. tem um talento claramente acima da média em matemática e pertence à
> elite dos "olímpicos"?
>
> 3) O Sr. não acha que o exibicionismo com estes problemas dificílimos
> acaba por alienar os alunos normais?
>
> 4) Qual a aplicabilidade na vida real de problemas de olimpíadas de
> matemática?
>
> Sds
>
> --
> Esta mensagem foi verificada pelo sistema de antivírus e
> acredita-se estar livre de perigo.

-- 
Esta mensagem foi verificada pelo sistema de antiv�rus e
 acredita-se estar livre de perigo.



[obm-l] Re: [obm-l] Re: [obm-l] Re: [obm-l] Re: [obm-l] polinômios

2017-11-28 Por tôpico Carlos Nehab
Oi, Mateus et alli

Eu cutuquei o Ralph porque há tempos ele colocou exatamente essa sua
explicação "vindo em defesa" de uma solução que eu havia postado de outro
problema". Rsrsr.
Achei importante explicitar esse detalhe pra galera.

Grande abraço
Nehab


Em 28 de novembro de 2017 12:07, Matheus Secco <matheusse...@gmail.com>
escreveu:

> Para ver que Q(x), basta ver que (x-a)(x-b)(x-c)(x-d) tem coeficiente
> lider 1 e ao fazer a divisão longa de P(x) por este polinomio com
> coeficiente lider 1, não há riscos de introduzir frações.
>
> Abs,
> Secco
>
> Em 28 de nov de 2017 11:58 AM, "Carlos Nehab" <carlos.ne...@gmail.com>
> escreveu:
>
> Oi, Ralph
>
> E o detalhe que Q(x) tem coeficientes inteiros..., "exprica prá nóis"!
>
> Abraços
> Nehab
>
> Em 27 de novembro de 2017 21:51, Ralph Teixeira <ralp...@gmail.com>
> escreveu:
>
>> Acho que eles queriam 4 raizes inteiras distintas.
>>
>> Neste caso, temos P(x)=(x-a)(x-b)(x-c)(x-d)Q(x) onde Q(x) tem
>> coeficientes inteiros e a,b,c,d sao as 4 raizes inteiras distintas.
>>
>> Se P(x)=2 tivesse raiz inteira, digamos, x=n, entao teriamos
>> P(n)=(n-a)(n-b)(n-c)(n-d)Q(n)=2. Mas entao n-a, n-b, n-c e n-d seriam 4
>> inteiros distintos cujo produto seria +-1 ou +-2, o que nao eh possivel.
>>
>> Abraco, Ralph.
>>
>> 2017-11-27 20:09 GMT-02:00 André Lauer <andre_la...@hotmail.com.br>:
>>
>>> Boa noite, preciso de ajuda no seguinte problema:
>>> Um polinômio P(x) tem coeficientes inteiros e admite quatro raízes
>>> inteiras. Prove que a equação P(x) = 2 não admite raízes inteiras.
>>>
>>> --
>>> Esta mensagem foi verificada pelo sistema de antivírus e
>>> acredita-se estar livre de perigo.
>>>
>>
>>
>> --
>> Esta mensagem foi verificada pelo sistema de antivírus e
>> acredita-se estar livre de perigo.
>
>
>
> --
> Esta mensagem foi verificada pelo sistema de antivírus e
> acredita-se estar livre de perigo.
>
>
>
> --
> Esta mensagem foi verificada pelo sistema de antivírus e
> acredita-se estar livre de perigo.

-- 
Esta mensagem foi verificada pelo sistema de antiv�rus e
 acredita-se estar livre de perigo.



[obm-l] Re: [obm-l] Re: [obm-l] polinômios

2017-11-28 Por tôpico Carlos Nehab
Oi, Ralph

E o detalhe que Q(x) tem coeficientes inteiros..., "exprica prá nóis"!

Abraços
Nehab

Em 27 de novembro de 2017 21:51, Ralph Teixeira 
escreveu:

> Acho que eles queriam 4 raizes inteiras distintas.
>
> Neste caso, temos P(x)=(x-a)(x-b)(x-c)(x-d)Q(x) onde Q(x) tem coeficientes
> inteiros e a,b,c,d sao as 4 raizes inteiras distintas.
>
> Se P(x)=2 tivesse raiz inteira, digamos, x=n, entao teriamos
> P(n)=(n-a)(n-b)(n-c)(n-d)Q(n)=2. Mas entao n-a, n-b, n-c e n-d seriam 4
> inteiros distintos cujo produto seria +-1 ou +-2, o que nao eh possivel.
>
> Abraco, Ralph.
>
> 2017-11-27 20:09 GMT-02:00 André Lauer :
>
>> Boa noite, preciso de ajuda no seguinte problema:
>> Um polinômio P(x) tem coeficientes inteiros e admite quatro raízes
>> inteiras. Prove que a equação P(x) = 2 não admite raízes inteiras.
>>
>> --
>> Esta mensagem foi verificada pelo sistema de antivírus e
>> acredita-se estar livre de perigo.
>>
>
>
> --
> Esta mensagem foi verificada pelo sistema de antivírus e
> acredita-se estar livre de perigo.

-- 
Esta mensagem foi verificada pelo sistema de antiv�rus e
 acredita-se estar livre de perigo.



[obm-l] Re: [obm-l] Lógica

2017-11-26 Por tôpico Carlos Nehab
Apenas corrigindo o detalhe...

Vamos lá:
As proposições
p: (qqsx)(se x é racional então y é irracional)
~p (não p): (há x)(x é racional *e* y é racional)
são (verdadeiras). FALSAS, de fato.

Em 26 de novembro de 2017 21:05, Carlos Nehab <carlos.ne...@gmail.com>
escreveu:

> Oi, Israel,
>
> Sua encrenca reside na falta de uso de quantificadores.
> A contrapositiva de uma proposição do tipo P implica Q é, como vc
> mencionou, ~Q implica ~P.
> Mas a proposição original a que vc se refere parece ser "se x é racional
> então y é irracional" mas não é, pois falta o uso dos quantificadores
> adequados.
> A proposição original é (qqsx)(x racional então y = x - 1/x é racional).
>
> Observe que, tecnicamente, a expressão "x = x" NÃO é uma proposição, não
> podendo a ela ser atribuído nenhum valor verdade.
> A proposição a que poderíamos atribuir valor verdade é (qqs x)( x = x).
> Essa sim é uma proposição e gostaríamos de dizer que ela é verdadeira.
>
> Vamos lá:
> As proposições
> p: (qqsx)(se x é racional então y é irracional)
> ~p (não p): (há x)(x é racional *e* y é racional)
> são verdadeiras.
>
> Mas faz sentido falar em contrapositiva a não ser para uma PROPOSIÇÃO do
> tipo "P implica Q".
> A proposição (qqs)[p(x) implica q(x)] não tem, tecnicamente,
> contrapositiva.
> Sua negação é "(existe x)[p(x) e ~q(x)]. E só.
>
> Abraços,
> Nehab
>
>
>
>
>
> Em 26 de novembro de 2017 20:28, Israel Meireles Chrisostomo <
> israelmchrisost...@gmail.com> escreveu:
>
>>
>> Seja y=x-1/x.A proposição: se x é racional então y é irracional, é uma
>> proposição claramente falsa.Mas se uma proposição é falsa, então sua
>> contra-positiva também é falsa.A contra-positiva dessa sentença é:  se y é
>> racional então x é irracional, e por ser a proposição contra-positiva da
>> primeira, então essa sentença também é falsa.Mas se essa sentença é falsa,
>> então se y é racional então x só pode ser racional.Agora veja que se
>> x=1+sqrt{2}, podemos ver claramente que y será racional, ou seja, o fato de
>> y ser racional não implicaria que x é racional.O que eu fiz de errado?
>> --
>> Israel Meireles Chrisostomo
>>
>> --
>> Esta mensagem foi verificada pelo sistema de antivírus e
>> acredita-se estar livre de perigo.
>
>
>

-- 
Esta mensagem foi verificada pelo sistema de antiv�rus e
 acredita-se estar livre de perigo.



[obm-l] Re: [obm-l] Lógica

2017-11-26 Por tôpico Carlos Nehab
Oi, Israel,

Sua encrenca reside na falta de uso de quantificadores.
A contrapositiva de uma proposição do tipo P implica Q é, como vc
mencionou, ~Q implica ~P.
Mas a proposição original a que vc se refere parece ser "se x é racional
então y é irracional" mas não é, pois falta o uso dos quantificadores
adequados.
A proposição original é (qqsx)(x racional então y = x - 1/x é racional).

Observe que, tecnicamente, a expressão "x = x" NÃO é uma proposição, não
podendo a ela ser atribuído nenhum valor verdade.
A proposição a que poderíamos atribuir valor verdade é (qqs x)( x = x).
Essa sim é uma proposição e gostaríamos de dizer que ela é verdadeira.

Vamos lá:
As proposições
p: (qqsx)(se x é racional então y é irracional)
~p (não p): (há x)(x é racional *e* y é racional)
são verdadeiras.

Mas faz sentido falar em contrapositiva a não ser para uma PROPOSIÇÃO do
tipo "P implica Q".
A proposição (qqs)[p(x) implica q(x)] não tem, tecnicamente, contrapositiva.
Sua negação é "(existe x)[p(x) e ~q(x)]. E só.

Abraços,
Nehab





Em 26 de novembro de 2017 20:28, Israel Meireles Chrisostomo <
israelmchrisost...@gmail.com> escreveu:

>
> Seja y=x-1/x.A proposição: se x é racional então y é irracional, é uma
> proposição claramente falsa.Mas se uma proposição é falsa, então sua
> contra-positiva também é falsa.A contra-positiva dessa sentença é:  se y é
> racional então x é irracional, e por ser a proposição contra-positiva da
> primeira, então essa sentença também é falsa.Mas se essa sentença é falsa,
> então se y é racional então x só pode ser racional.Agora veja que se
> x=1+sqrt{2}, podemos ver claramente que y será racional, ou seja, o fato de
> y ser racional não implicaria que x é racional.O que eu fiz de errado?
> --
> Israel Meireles Chrisostomo
>
> --
> Esta mensagem foi verificada pelo sistema de antivírus e
> acredita-se estar livre de perigo.

-- 
Esta mensagem foi verificada pelo sistema de antiv�rus e
 acredita-se estar livre de perigo.



[obm-l] Re: [obm-l] Problema de álgebra

2017-09-15 Por tôpico Carlos Nehab
Oi, Leonardo (e Ralph)

Resolvi postar meu "rabisco de tentativa de solução" pois acho (e com
certeza Ralph tb) que isso enriquece o aprendizado da gurizada (sorry pelo
gurizada, mas me formei em 1969...).

Fiz o seguinte:
(Supondo numa primeira abordagem que x, y e z fossem >= -1, prá ver onde
isso poderia parar).

Calculei MG e MA entre (1+x), (1+y) e (1+z).

A razão desse aparente coelho da cartola, Leonardo, é que o tal do x+y+z =
1, o xy+xz+yz e o xyz do seu enunciado me deram uma coceira num "produtinho
não tão notável, mas útil, que já matou inúmeros problemas, qual seja:

(1+x)(1+y)(1+z) = 1 + (x+y+z) + (xy+yz+xz) + xyz, que, nesse seu caso, vale
2 + a + xyz

Então, prosseguindo:

MA >= MG, acarreta 4/3 >= (2 + a + xyz)^1/3, ou seja, xyz <= (10/27 - a),
que aliás, é positivo, pois  a < 1/3).

Mas esse rabisco não funciona pois não conseguimos garantir que o xyz
atinge o valor (10/27) - a, até porque isso só ocorreria se x = y = z o que
não é o caso, conforme a criativa solução do Ralph.

Abs de um colega mais velho que a lista (rsrsrs)...
Nehab

Em 15 de setembro de 2017 15:13, Leonardo Joau 
escreveu:

> Dados os reais x, y,z, tais que:
>
> x+y+z = 1
>
> xy+xz+yz = a  0
> Calcule o max{xyz} em função de a.
>
>
> Att,
> Leonardo Joau
>
> --
> Esta mensagem foi verificada pelo sistema de antivírus e
> acredita-se estar livre de perigo.

-- 
Esta mensagem foi verificada pelo sistema de antiv�rus e
 acredita-se estar livre de perigo.



[obm-l] Re: [obm-l] fatoração

2017-09-12 Por tôpico Carlos Nehab
Oi,
x3 + x2y + x2y + x2y + xy2 + xy2 + xy2 + y3
= (x3 + x2y) + 2(x2y+xy2) + (xy2 + y3)
=  x2*(x+y)* + 2xy*(x+y)* + y2*(x+y) *
= (x2+2xy+y2)(x+y) = (x+y)3...
The end...

Em 12 de setembro de 2017 14:23,  escreveu:

> Meus amigos, por favor,  como fatorar  (agrupando!?) x^3 + 3x^2y + 3xy^2 +
> y^3 e chegar em (x+y)^3 ?
>
> (x+y)^3=x^3 + 3x^2y+3xy^2+y^3
>
> Perdoem –me !
>
> Abraços
>
> Hermann
>
> --
> Esta mensagem foi verificada pelo sistema de antivírus e
> acredita-se estar livre de perigo.
>

-- 
Esta mensagem foi verificada pelo sistema de antiv�rus e
 acredita-se estar livre de perigo.



Re: [obm-l] Fibonacci teoria dos numeros

2017-09-04 Por tôpico Carlos Nehab
Oi, Douglas.

Acho que o mdc entre Fibbonaccis consecutivos é sempre 1...

Nehab


Livre
de vírus. www.avast.com
.
<#m_2006884623661450834_DAB4FAD8-2DD7-40BB-A1B8-4E2AA1F9FDF2>

Em 4 de setembro de 2017 07:24, Anderson Torres <
torres.anderson...@gmail.com> escreveu:

> Em 31 de agosto de 2017 16:30, Douglas Oliveira de Lima
>  escreveu:
> > Olá, como posso mostrar que para algum inteiro e positivo n, existe um
> > número de Fibonacci que é múltiplo de n?
>
> Casa dos Pombos! Maybe?
>
> Bem, pegue os pares de Fibonaccis consecutivos, (F0, F1), (F1, F2),
> (F2, F5),... módulo M.
>
> Por PCP, dois deles , digamos (Fk, F(k+1)) e (F(k+j),F(k+j+1)) serão
> iguais.
>
> Assim, Fk=F(k+j) e F(k+1)=F(k+j+1).
>
> Mas aí, F(k+1)-F(k)=F(k+j+1)-F(k+j) e portanto F(k-1) = F(k+j-1).
>
> Prosseguindo dessa forma, chegaremos em F(j)=F(0)=0.
>
>
>
> >
> > Douglas Oliveira.
> >
> >
> > --
> > Esta mensagem foi verificada pelo sistema de antivírus e
> > acredita-se estar livre de perigo.
>
> --
> Esta mensagem foi verificada pelo sistema de antivírus e
>  acredita-se estar livre de perigo.
>
>
> =
> Instru�ões para entrar na lista, sair da lista e usar a lista em
> http://www.mat.puc-rio.br/~obmlistas/obm-l.html
> =
>

-- 
Esta mensagem foi verificada pelo sistema de antiv�rus e
 acredita-se estar livre de perigo.



Re: [obm-l] Leningrad Olympiads

2017-08-20 Por tôpico Carlos Nehab
Tem aqui...

https://www.elephant-ads.com/LP_TA/index.cfm?T=437235

Abs
Nehab


Livre
de vírus. www.avast.com
.
<#DAB4FAD8-2DD7-40BB-A1B8-4E2AA1F9FDF2>

Em 19 de agosto de 2017 19:01, Ricardo Leão 
escreveu:

> Eu tenho procurado o seguinte livro:
>
> - Fomin, D; Kirichenko, A; *Leningrad Mathematical Olympiads 1987-1991*
> (1994)
>
> Alguém aí sabe onde eu posso encontrar esse livro???
>
> --
> Esta mensagem foi verificada pelo sistema de antivírus e
> acredita-se estar livre de perigo.

-- 
Esta mensagem foi verificada pelo sistema de antiv�rus e
 acredita-se estar livre de perigo.



Re: [obm-l] Garrafa de Klein

2017-08-14 Por tôpico Carlos Nehab
Facilmente. Eu tenho uma de vidro. Vc pode demonstrar a construcao com
aqueles trançadinhos amarelos onde se comprava laranjas na feira.

Abs

Em 13/08/2017 20:17, "Luiz Antonio Rodrigues" 
escreveu:

> Olá, pessoal!
> Boa noite!
> Eu tenho uma dúvida desde os tempos da faculdade... Uma garrafa de Klein
> pode ser construída? Eu consultei a Internet e me confundi ainda mais...
> Um abraço para todos!
> Luiz
>
> --
> Esta mensagem foi verificada pelo sistema de antivírus e
> acredita-se estar livre de perigo.

-- 
Esta mensagem foi verificada pelo sistema de antiv�rus e
 acredita-se estar livre de perigo.



[obm-l] Re: [obm-l] Teoria dos números

2017-07-06 Por tôpico Carlos Nehab
Oi, Douglas,

Esse "abc" é a x b x c (produto) ou o inteiro de algarismos a, b e c
(100a+10b+c)?

Abs
Nehab


Livre
de vírus. www.avast.com
.
<#DAB4FAD8-2DD7-40BB-A1B8-4E2AA1F9FDF2>

Em 6 de julho de 2017 14:03, Douglas Oliveira de Lima <
profdouglaso.del...@gmail.com> escreveu:

> Encontrar todos os inteiros positivos a,b e c tais que a^b+b^c=abc.
>
> --
> Esta mensagem foi verificada pelo sistema de antivírus e
> acredita-se estar livre de perigo.

-- 
Esta mensagem foi verificada pelo sistema de antiv�rus e
 acredita-se estar livre de perigo.



Re: [obm-l] Fatorial

2016-11-03 Por tôpico Carlos Nehab
Oi, Israel.

Tô com preguiça, mas se meus neurônios permanecem intactos, vc pode se
convencer disso, percebendo que se os n inteiros positivos consecutivos são
p, p+1, ... p+n-1, o quociente desse produto por n! corresponde exatamente
à quantidade de subconjuntos que se podem formar de n objetos escolhidos
dentre n+p-1 objetos dados... Logo, é um inteiro.  Ou seja é o número
combinatório "combinação de  n+p-1 objetos n a n".

Abraços
Nehab

Em 3 de novembro de 2016 12:59, Israel Meireles Chrisostomo <
israelmchrisost...@gmail.com> escreveu:

> Olá pessoal como posso provar que n! divide o produto de quaisquer n
> inteiros consecutivos
>
> --
> Esta mensagem foi verificada pelo sistema de antivírus e
> acredita-se estar livre de perigo.

-- 
Esta mensagem foi verificada pelo sistema de antiv�rus e
 acredita-se estar livre de perigo.



Re: [obm-l] Teorema de Varignon

2016-03-19 Por tôpico Carlos Nehab
Titio google nao respondeu?
Em 18/03/2016 11:57, "Luís"  escreveu:

> Sauda,c~oes,
>
>
> O teorema de Varignon é bem conhecido: os pontos médios dos lados
>
> de um quadrilátero formam um paralelogramo.
>
>
> Alguém conhece uma referência em português que o demostra ?
>
> Não preciso da demonstração, só a citação.
>
>
> Penso ter visto algo a respeito na RPM, Eureka, publicações do
>
> IMPA ou livro de Geometria do Wagner/Morgado.
>
>
> Abs,
>
> Luís
>
>
>
> --
> Esta mensagem foi verificada pelo sistema de antivírus e
> acredita-se estar livre de perigo.
>

-- 
Esta mensagem foi verificada pelo sistema de antiv�rus e
 acredita-se estar livre de perigo.



[obm-l] Re: [obm-l] Re: [obm-l] Cálculo 2 Serge Lang

2015-09-17 Por tôpico Carlos Nehab
E pdf? Quando vc escrever um livro? Como vai ser?
Nehab
Em 16/09/2015 23:55, "Bernardo Freitas Paulo da Costa" <
bernardo...@gmail.com> escreveu:

> 2015-09-16 21:04 GMT-03:00 Israel Meireles Chrisostomo
> :
> >
> > Alguém sabe onde encontro na net o pdf do livro Cálculo 2 do Serge Lang?
>
> Israel, esta lista é para discutir problemas olímpicos e relacionados,
> não o que você pediu. E não sei se o Lang de Cálculo seria uma
> referência tão importante e rara assim para justificar a sua mensagem.
>
> --
> Bernardo Freitas Paulo da Costa
>
> --
> Esta mensagem foi verificada pelo sistema de antivírus e
>  acredita-se estar livre de perigo.
>
>
> =
> Instru�ões para entrar na lista, sair da lista e usar a lista em
> http://www.mat.puc-rio.br/~obmlistas/obm-l.html
> =
>

-- 
Esta mensagem foi verificada pelo sistema de antiv�rus e
 acredita-se estar livre de perigo.



Re: [obm-l] Irracionais

2015-08-31 Por tôpico Carlos Nehab
Serve 0 radianos? Rsrsrs.

Abs
Nehab

Em 1 de setembro de 2015 00:53, Israel Meireles Chrisostomo <
israelmchrisost...@gmail.com> escreveu:

> Existe algum ângulo (em radianos) que seja racional e cujo o cosseno seja
> um número racional?
>
> --
> Esta mensagem foi verificada pelo sistema de antivírus e
> acredita-se estar livre de perigo.

-- 
Esta mensagem foi verificada pelo sistema de antiv�rus e
 acredita-se estar livre de perigo.



[obm-l] Re: [obm-l] Análise Combinatória

2015-08-13 Por tôpico Carlos Nehab
K! Esse é o tipo de questão indigna, para o ENEM. Contexto inadequado!
Kkkk.

Abs
Nehab
Em 11/08/2015 10:22, Pedro Costa npc1...@gmail.com escreveu:

 Uma aranha tem uma meia e um sapato paracada um de seus oito pés. De
 quantas maneiras diferentes

 a aranha pode se calçar admitindo que a meia tem que ser colocada antes do
 sapato?


 --
 [image: Avast logo] https://www.avast.com/antivirus

 Este email foi escaneado pelo Avast antivírus.
 www.avast.com https://www.avast.com/antivirus


 --
 Esta mensagem foi verificada pelo sistema de antivírus e
 acredita-se estar livre de perigo.


-- 
Esta mensagem foi verificada pelo sistema de antiv�rus e
 acredita-se estar livre de perigo.



[obm-l] Re: [obm-l] Limite Com 3 Variáveis

2015-07-27 Por tôpico Carlos Nehab
Oi Daniel,
Brinque com as variáveis x, y e z percorrendo sequências do tipo 1/n,
1/n^2 etc e vc verá que os limites , caso existissem, não seriam únicos.
Abs
Nehab
Em 25/07/2015 23:07, Daniel Rocha daniel.rocha@gmail.com escreveu:

 Olá a todos,

 Como eu posso mostrar que os Limites abaixo NÃO EXISTEM ???

  1)lim   X^2 + Y^2 - Z^2 / X^2 + Y^2 + Z^2
(x,y,z)-(0,0,0)

   2)   lim   X^4 +Y(X^3) + (Z^2)(X^2) / X^4 + Y^4 + Z^4
(x,y,z)-(0,0,0)

 Eu agradeço muito a quem me responder.


 --
 Esta mensagem foi verificada pelo sistema de antivírus e
 acredita-se estar livre de perigo.

-- 
Esta mensagem foi verificada pelo sistema de antiv�rus e
 acredita-se estar livre de perigo.



[obm-l] Re: [obm-l] Polinômios

2015-07-09 Por tôpico Carlos Nehab
Simples, Gabriel.

A solução dele da página 260 está errada e a sua certa. 
Fica frio.
Tá estudando num ótimo livro.

Abs Nehab

Em 8 de julho de 2015 22:07, Gabriel Tostes gtos...@icloud.com escreveu:

 Ache o resto de x^100 -2.x^51 + 1 na divisao por x^2 - 1.
 Eu nao entendo por que o resto eh 4x nao  -2x + 2
 Se fizer x=1 nao fica a + b = 0 ? E x=-1 -a+b=4 r(x) = ax + b
 Esse exercicio ta no livro do Engel, problem solving strategies.


 --
 Esta mensagem foi verificada pelo sistema de antivírus e
  acredita-se estar livre de perigo.

 =
 Instruções para entrar na lista, sair da lista e usar a lista em
 http://www.mat.puc-rio.br/~obmlistas/obm-l.html
 =


-- 
Esta mensagem foi verificada pelo sistema de antiv�rus e
 acredita-se estar livre de perigo.



[obm-l] Re: [obm-l] Re: [obm-l] Combinatória

2015-06-14 Por tôpico Carlos Nehab
Oi, Mateus,

Muito feliz com sua chegada por essas bandas.
Diminuiremos nossa idade média (ufa) e aumentaremos relevantemente o número
de neurônios competentes (outro ufa).

Grande abraço,
Nehab

Em 14 de junho de 2015 12:49, Matheus Secco matheusse...@gmail.com
escreveu:

 Oi Marcone, associe um sinal de + a um livro escolhido e um sinal de - a
 um livro não escolhido. Devemos colocar então 5 sinais de + e 7 sinais de
 -, sem que haja dois sinais de + juntos.

 Coloque os sinais de -`s primeiro. Eles gerarão 8 espaços e devemos
 colocar no máximo um sinal de + nestes espaços. Basta assim escolher 5 dos
 8 espaços criados, o que pode ser feito de C(8,5) = 56 maneiras.

 Abraços,

 Matheus Secco

 2015-06-14 10:16 GMT-03:00 marcone augusto araújo borges 
 marconeborge...@hotmail.com:

 Uma prateleira contém 12 livros. De quantas maneiras podemos escolher 5
 deles
 de modo que dois dos livros escolhidos nunca fiquem um ao lado do outro?



 --
 Esta mensagem foi verificada pelo sistema de antivírus e
 acredita-se estar livre de perigo.



 --
 Esta mensagem foi verificada pelo sistema de antivírus e
 acredita-se estar livre de perigo.

-- 
Esta mensagem foi verificada pelo sistema de antiv�rus e
 acredita-se estar livre de perigo.



[obm-l] Re: [obm-l] Re: [obm-l] Só compartilhando

2015-05-24 Por tôpico Carlos Nehab
Rsrsrs.
Vc pensou em dois triângulos equilátero também, cuja razão vale.?
Possivelmente faltou no enunciado que os triângulos devem ter os três lados
desiguais.
Abs
Nehab
Abs
Em 23/05/2015 21:53, Bernardo Freitas Paulo da Costa 
bernardo...@gmail.com escreveu:

 2015-05-23 14:55 GMT-03:00 marcone augusto araújo borges
 marconeborge...@hotmail.com:
  Determine 2 triângulos não congruentes tais que 5 elementos de um deles
  sejam congruentes a 5 elementos do outro.

 O que conta como elementos? Eu tenho uma solução com MUITO mais...
 (e que ainda dá lados inteiros)

  Os lados dos triângulos podem se números inteiros?

 Abraços,
 --
 Bernardo Freitas Paulo da Costa

 --
 Esta mensagem foi verificada pelo sistema de antivírus e
  acredita-se estar livre de perigo.


 =
 Instru�ões para entrar na lista, sair da lista e usar a lista em
 http://www.mat.puc-rio.br/~obmlistas/obm-l.html
 =


-- 
Esta mensagem foi verificada pelo sistema de antiv�rus e
 acredita-se estar livre de perigo.



Re: [obm-l] Quadrados perfeitos

2015-05-17 Por tôpico Carlos Nehab
Oi Marconi.
Pq qualquer cara depois do 1444 qdo dividido por 4 dá um ímpar do tipo
36111 e esse ímpar pra ser quadrado de um sujeitinho tb ímpar deveria
deixar resto 1 qdo dividido por 4. E não deixa, pois 36...110 qdo dividido
por 4 deixa resto 2.

Abs
Nehab
Em 15/05/2015 23:47, marcone augusto araújo borges 
marconeborge...@hotmail.com escreveu:

 Mostre que entre os números da forma 14,144,1444,144...4,... os únicos que
 são quadrados perfeitos
 são 144 e 1444

 --
 Esta mensagem foi verificada pelo sistema de antivírus e
 acredita-se estar livre de perigo.


-- 
Esta mensagem foi verificada pelo sistema de antiv�rus e
 acredita-se estar livre de perigo.



Re: [obm-l] Funções

2014-03-10 Por tôpico Carlos Nehab
Profmat...
Nehab

Enviado via iPhone

Em 10/03/2014, às 08:00, marcone augusto araújo borges 
marconeborge...@hotmail.com escreveu:

 Sejam f e g duas funções f: X -- Y e g: Y-- X.Prove que
 
 a) Se gof é injetiva,então f é injetiva
 
 b) Se fog é sobrejetiva,então g é sobrejetiva
 
 -- 
 Esta mensagem foi verificada pelo sistema de antivírus e 
 acredita-se estar livre de perigo.

-- 
Esta mensagem foi verificada pelo sistema de antiv�rus e
 acredita-se estar livre de perigo.



Re: [obm-l] Re: [obm-l] Re: [obm-l] Polinômio

2014-03-09 Por tôpico Carlos Nehab

Oi, Bernardo (e demais colegas...)

Toda razão pras observações do Bernardo!
É ótimo tê-lo no pé da gente. Sempre atento (há décadas - rsrsrs).
Minha suposta solução NÃO resolve o problema proposto pelo Marcone.
Da proxima vez serei menos apressado...

Obrigado e abraços,
Nehab

On 08/03/2014 16:19, Bernardo Freitas Paulo da Costa wrote:

2014-03-08 14:41 GMT-03:00 Cláudio Gustavo claudiog...@yahoo.com.br:

Num polinômio com coeficientes inteiros, ao se substituírem dois valores quaisquer a e b do 
domínio e subtraindo as expressões de p(b) e p(a) eh possível colocar o fator b-a em evidencia. Observando 
que o outro fator que multiplica b-a continua sendo inteiro, tem-se que (p(b)-p(a))/(b-a) eh inteiro e que 
b-a divide p(b)-p(a).

Eu não contestei a sua solução, Cláudio. O meu problema é com a
solução do Nehab. Continuo sem ver como usar a expressão p(x) =
(x-1)(x-2)(x-3)Q(x) + ax^2 + bx + c ajuda a resolver a questão. A
divisão euclidiana que ele faz (conforme a outra mensagem dele na
lista) não garante que Q(x) tem coeficientes inteiros.

Abraços,



---
Este email está limpo de vírus e malwares porque a proteção do avast! Antivírus 
está ativa.
http://www.avast.com


--
Esta mensagem foi verificada pelo sistema de antivírus e
acredita-se estar livre de perigo.

=
Instruções para entrar na lista, sair da lista e usar a lista em
http://www.mat.puc-rio.br/~obmlistas/obm-l.html
=


Re: [obm-l] Respostas que não chegam

2014-03-08 Por tôpico Carlos Nehab
Oi Marcone, 
Porque qdo dividimos im polinômio P(x) por um polinômio do terceiro grau (o 
produto dos três fatores) obtemos um quociente q(x) e um resto de grau no 
máximo 2.

Abs
Nehab

Enviado via iPhone

Em 07/03/2014, às 21:58, marcone augusto araújo borges 
marconeborge...@hotmail.com escreveu:

 Ultimamente quando respondo a alguma mensagem,nada.
 Sobre´´polinômio´´,agradeço ao Nehab,mas não sei porque
 podemos escrever p(x) do jeito que ele propôs.
 
 -- 
 Esta mensagem foi verificada pelo sistema de antivírus e 
 acredita-se estar livre de perigo.

-- 
Esta mensagem foi verificada pelo sistema de antiv�rus e
 acredita-se estar livre de perigo.



Re: [obm-l] Polinômio

2014-03-07 Por tôpico Carlos Nehab
Faça p(x) : (x-1)(x-2)(x-3)Q(x) mais ax2 mais bx mais c e aplique as três 
condições.
Nehab

Enviado via iPhone

Em 07/03/2014, às 11:55, marcone augusto araújo borges 
marconeborge...@hotmail.com escreveu:

 Mostre que não existe um polinômio p(x) com coeficientes inteiros tal que
 p(1) = 2,p(2) = 3 e p(3) = 5
 
 -- 
 Esta mensagem foi verificada pelo sistema de antivírus e 
 acredita-se estar livre de perigo.

-- 
Esta mensagem foi verificada pelo sistema de antiv�rus e
 acredita-se estar livre de perigo.



Re: [obm-l] Teorema da Incompletude de Godel

2014-02-02 Por tôpico Carlos Nehab

Oi, Luiz

Mande sua análise.
Você é da patota da Ciência da Computação, Matemática ou Filosofia?

Abs
Nehab

On 01/02/2014 23:46, luiz silva wrote:
É que eu estou querendo saber se tenho como formalizar uma análise que 
fiz.


Abs
Felipe


Em Sábado, 1 de Fevereiro de 2014 23:21, Francisco Barreto 
costadutrabarr...@gmail.com escreveu:
Não sou especialista, perdoe-me. Meu nome é Francisco Costa Barreto e 
este assunto me interessa. Estou acompanhando, quem sabe não torno-me 
útil em alguns dias para ajudá-lo neste aspecto, em tempo.
Eu costumo dizer incompleteza, mas podem me acusar de ser um 
Stickler neste caso.

=)
Abraços,
Francisco.


2014-02-01 luiz silva luizfelipec...@yahoo.com.br 
mailto:luizfelipec...@yahoo.com.br:


Pessoal,

Alguem aqui é especialista em logica-matematica, e conhece bem o
teorema de godel?

Abs
Felipe

-- 
Esta mensagem foi verificada pelo sistema de antivírus e

acredita-se estar livre de perigo.



--
Esta mensagem foi verificada pelo sistema de antivírus e
acredita-se estar livre de perigo.



--
Esta mensagem foi verificada pelo sistema de antivírus e
acredita-se estar livre de perigo. 




---
Este email está limpo de vírus e malwares porque a proteção do avast! Antivírus 
está ativa.
http://www.avast.com

--
Esta mensagem foi verificada pelo sistema de antivírus e
acredita-se estar livre de perigo.



Re: [obm-l] Apostila de Desenho 2 Impacto OFF TOPIC

2013-12-10 Por tôpico Carlos Nehab
Tenho acompanhado a gentileza do colega c as apostilas do Brandão.
Sugiro que algum dos agraciados c a gentileza escaneie a apostila e a pendure 
em algum lugar, por exemplo no scribd.

Embora eu tivesse sido prof do Brandão no IME e depois seu colega no 
magisterio, infelizmente não tenho suas apostilas...

Abs Nehab

Enviado via iPhone

Em 10/12/2013, às 15:09, Jeferson Almir jefersonram...@gmail.com escreveu:

 Oi João trabalho nos dois programas do governo referente a olimpiada de 
 matemática o POT e OBMEP aqui em fortaleza do meu belo ceará  e há tempos 
 procuro um material de desenho geométrico para trabalhar com os meninos pois 
 sinto a carência deles em problemas que requer construçoes geométricas e 
 reforço o meu interesse sobre o tão recomendado material. Jeferson Almir
 
 Em terça-feira, 10 de dezembro de 2013, Graciliano Antonio Damazo escreveu:
 Boa tarde, 
 acabei de receber minha apostila.
 Obrigado.
 Graciliano
 
 
 Em Terça-feira, 10 de Dezembro de 2013 10:51, Mauricio de Araujo 
 mauricio.de.ara...@gmail.com escreveu:
 Apostila recebida, muito obrigado!!!
 
 
 2013/12/4 Jefferson Franca jeffma...@yahoo.com.br
 Muito obrigado
 
 
 Em Terça-feira, 3 de Dezembro de 2013 17:36, jjun...@fazenda.ms.gov.br 
 jjun...@fazenda.ms.gov.br escreveu:
 Prezados amigos, 
 
 Não há necessidade de reembolso. 
 Os valores gastos não foram elevados. 
 Vocês serão úteis a vários e distintos jovens pelo país, e, assim, eu 
 também, indiretamente...
 
 Fraternalmente, João.
 
 
 
 - Mensagem Original -
 De:
 obm-l@mat.puc-rio.br
 
 Para:
 obm-l@mat.puc-rio.br obm-l@mat.puc-rio.br
 Cópia:~e
 
 Enviado:
 Tue, 3 Dec 2013 11:57:08 -0800 (PST)
 Assunto:
 Re: [obm-l] Apostila de Desenho 2 Impacto OFF TOPIC
 
 
 Obrigado João pelo envio do material.
 Me informe o numero e agencia de sua conta para que eu possa lhe pagar os 
 custos de xerox e correio (e mais tempo perdido para fazer o serviço).
 Mais uma vez agradeço e o que precisar é só avisar.
 Abraços
 Graciliano
 
 
 Em Terça-feira, 3 de Dezembro de 2013 15:43, Carlos Victor 
 victorcar...@globo.com escreveu:
 Obrigado João,
 
 Envie-me a sua conta bancária para depósito, ok ?
 
 Também posso lhe enviar o custo pelo correio.
 
 Agradeço e fico a disposição para o que precisares.
 
 Abraços
 
 Carlos Victor
 
 
 Em 1 de dezembro de 2013 17:54, jjun...@fazenda.ms.gov.br escreveu:
  Senhores: 
 
 Ontem (sábado), por volta das 15h em Campo Grande, foi enviada cópia da 
 Apostila 2 de Desenho do IMPACTO, uma ao senhor Carlos Victor (Nilópolis - 
 RJ), e outra a Graciliano Antônio Damazo (Penápolis - SP).
 
 ATT.
 João (Campo Grande - MS)
 
 
 -- 
 Esta mensagem foi verificada pelo sistema de antivírus e 
 acredita-se estar livre de perigo.
 
 -- 
 Esta mensagem foi verificada pelo sistema de antivírus e 
 acredita-se estar livre de perigo.

-- 
Esta mensagem foi verificada pelo sistema de antiv�rus e
 acredita-se estar livre de perigo.



Re: [obm-l] Apostila de Desenho 2 Impacto OFF TOPIC

2013-12-10 Por tôpico Carlos Nehab
Caros colegas,

Se alguém do Rio tiver a apostila eu mando buscar, escanteio e penduro no 
Scribd com prazer.
Abs
Nehab

Enviado via iPhone

Em 10/12/2013, às 17:30, jjun...@fazenda.ms.gov.br escreveu:

 Ótima ideia, se alguém se puder... eu agradeceria
 
 - Mensagem Original -
 De:
 obm-l@mat.puc-rio.br
 
 Para:
 obm-l@mat.puc-rio.br obm-l@mat.puc-rio.br
 Cópia:
 
 Enviado:
 Tue, 10 Dec 2013 16:24:57 -0200
 Assunto:
 Re: [obm-l] Apostila de Desenho 2 Impacto OFF TOPIC
 
 
 Tenho acompanhado a gentileza do colega c as apostilas do Brandão.
 Sugiro que algum dos agraciados c a gentileza escaneie a apostila e a pendure 
 em algum lugar, por exemplo no scribd.
 
 Embora eu tivesse sido prof do Brandão no IME e depois seu colega no 
 magisterio, infelizmente não tenho suas apostilas...
 
 Abs Nehab
 
 Enviado via iPhone
 
 Em 10/12/2013, Ã s 15:09, Jeferson Almir jefersonram...@gmail.com escreveu:
 
 Oi João trabalho nos dois programas do governo referente a olimpiada de 
 matemática o POT e OBMEP aqui em fortaleza do meu belo ceará  e há 
 tempos procuro um material de desenho geométrico para trabalhar com os 
 meninos pois sinto a carência deles em problemas que requer construçoes 
 geométricas e reforço o meu interesse sobre o tão recomendado material. 
 Jeferson Almir
 
 Em terça-feira, 10 de dezembro de 2013, Graciliano Antonio Damazo escreveu:
 Boa tarde, 
 acabei de receber minha apostila.
 Obrigado.
 Graciliano
 
 
 Em Terça-feira, 10 de Dezembro de 2013 10:51, Mauricio de Araujo 
 mauricio.de.ara...@gmail.com escreveu:
 Apostila recebida, muito obrigado!!!
 
 
 2013/12/4 Jefferson Franca jeffma...@yahoo.com.br
 Muito obrigado
 
 
 Em Terça-feira, 3 de Dezembro de 2013 17:36, jjun...@fazenda.ms.gov.br 
 jjun...@fazenda.ms.gov.br escreveu:
 Prezados amigos, 
 
 Não há necessidade de reembolso. 
 Os valores gastos não foram elevados. 
 Vocês serão úteis a vários e distintos jovens pelo país, e, assim, eu 
 também, indiretamente...
 
 Fraternalmente, João.
 
 
 
 - Mensagem Original -
 De:
 obm-l@mat.puc-rio.br
 
 Para:
 obm-l@mat.puc-rio.br obm-l@mat.puc-rio.br
 Cópia:~e
 
 Enviado:
 Tue, 3 Dec 2013 11:57:08 -0800 (PST)
 Assunto:
 Re: [obm-l] Apostila de Desenho 2 Impacto OFF TOPIC
 
 
 Obrigado João pelo envio do material.
 Me informe o numero e agencia de sua conta para que eu possa lhe pagar os 
 custos de xerox e correio (e mais tempo perdido para fazer o serviço).
 Mais uma vez agradeço e o que precisar é só avisar.
 Abraços
 Graciliano
 
 
 Em Terça-feira, 3 de Dezembro de 2013 15:43, Carlos Victor 
 victorcar...@globo.com escreveu:
 Obrigado João,
 
 Envie-me a sua conta bancária para depósito, ok ?
 
 Também posso lhe enviar o custo pelo correio.
 
 Agradeço e fico a disposição para o que precisares.
 
 Abraços
 
 Carlos Victor
 
 
 Em 1 de dezembro de 2013 17:54, jjun...@fazenda.ms.gov.br escreveu:
 Â Senhores:Â 
 
 Ontem (sábado), por volta das 15h em Campo Grande, foi enviada cópia da 
 Apostila 2 de Desenho do IMPACTO, uma ao senhor Carlos Victor (Nilópolis - 
 RJ), e outra a Graciliano Antônio Damazo (Penápolis - SP).
 
 ATT.
 João (Campo Grande - MS)
 
 
 -- 
 Esta mensagem foi verificada pelo sistema de antivírus e 
 acredita-se estar livre de perigo.
 
 -- 
 Esta mensagem foi verificada pelo sistema de antivírus e 
 acredita-se estar livre de perigo.
 
 -- 
 Esta mensagem foi verificada pelo sistema de antivrus e 
 acredita-se estar livre de perigo.
 
 -- 
 Esta mensagem foi verificada pelo sistema de antivírus e 
 acredita-se estar livre de perigo.

-- 
Esta mensagem foi verificada pelo sistema de antiv�rus e
 acredita-se estar livre de perigo.



[obm-l] Re: [obm-l] Enc: Problema - Contagem de Triplos Pitagóricos

2012-11-07 Por tôpico Carlos Nehab

Oi, Luis,

Só para lembrar que a hipotenusa de um pitagórico primitivo não pode ser 
múltiplo de 4.

Logo, você está falando apenas em catetos da forma 4n.
E, neste caso, basta analisar as situações de seu cateto ser 4pq ou 
seja, contar os divisores de n primos com n... Clássico.

Mas sua solução não faz nenhuma referênca a triplas pitagóricas!
Não entendi seu argumento.

Abraços
Nehab

Em 06/11/2012 15:46, luiz silva escreveu:


Onde temos n, considerar n+1.
Abs
Felipe

- Mensagem encaminhada -
*De:* luiz silva luizfelipec...@yahoo.com.br
*Para:* Matematica Lista obm-l@mat.puc-rio.br
*Enviadas:* Terça-feira, 6 de Novembro de 2012 16:23
*Assunto:* Problema - Contagem de Triplos Pitagóricos

Pessoal,
Eu pensei em um problema, e fiz a seguinte solução (nao lembro se ja 
posei aqui) :
Dado um numero par da forma 4n, quantos são os triplos pitagoricos 
primitivos que posseum este numero como seu elemento ?

Este número N será da forma : 2. 2^a . P_1 ^b1 . P_2 ^b2 P_n ^bn
Dessa forma, podemos contar as soluções, considerando somente os primos.
Q = C^n _1 + C^n _2 + C^n _3 +...+ C^n _n
 Está correta esta solução ?






Re: [obm-l] Ajuda em geometria

2012-10-17 Por tôpico Carlos Nehab

Oi, Ralph,

Sem tempo para escrever mas com tempo para ler.
Embora você não precise de elogio, bela solução!

Abraços
Nehab

Em 17/10/2012 14:55, Ralph Teixeira escreveu:
Note que, dadas a soma A e o produto B de dois números, eles ficam 
determinados a menos de ordem -- afinal, eles são as raízes da 
quadrática x^2-Ax+B=0.


Então, neste caso, seja A=a+ha=b+hb=c+hc e B=aha=bhb=chc=2S. Então a e 
ha são as raízes de x^2-Ax+B=0, assim como b e hb, e c e hc. Em suma, 
a, b, c, assumem apenas (no máximo) dois valores (repetidos 3 vezes 
cada) -- o triângulo já tem que ser isósceles! Usemos, sem perder 
generalidade, que a=b.


Agora, suponha por contradição que o triângulo não é equilátero. Então 
a=b=hc. Mas isto é absurdo -- a e b são lados saindo de C, ao menos um 
deles tem que ser estritamente maior que hc, que é a MENOR distância 
de C até AB.


Abraço,
 Ralph

On Wed, Oct 17, 2012 at 8:52 AM, marcone augusto araújo borges 
marconeborge...@hotmail.com mailto:marconeborge...@hotmail.com wrote:


Seja um triangulo ABC, a,b,c as medidas do lados BC,AC e
AB,respectivamente e ha, hb e hc as alturas do triangulo.
Se a + ha = b + hb = c + hc,prove que ABC é equilatero.






[obm-l] Silêncio total?

2012-06-24 Por tôpico Carlos Nehab

Neguinho tá segurando a Lista por causa das Olimpíadas?
Nenhuma mensagem nos últimos dias?

Nehab
=
Instruções para entrar na lista, sair da lista e usar a lista em
http://www.mat.puc-rio.br/~obmlistas/obm-l.html
=


Re: [obm-l] RE: [obm-l] Silêncio total?

2012-06-24 Por tôpico Carlos Nehab

Mas já que nossas mensagens chegaram, desentupiu o servidor!
Acho que a turma tá mesmo é de férias...
Aguardemos...
Nehab

Em 24/06/2012 11:47, Felippe Coulbert Balbi escreveu:

Percebe a mesma coisa Nehab.

 Coulbert

 Date: Sun, 24 Jun 2012 07:17:06 -0300
 From: carlos.ne...@gmail.com
 To: obm-l@mat.puc-rio.br
 Subject: [obm-l] Silêncio total?

 Neguinho tá segurando a Lista por causa das Olimpíadas?
 Nenhuma mensagem nos últimos dias?

 Nehab
 
=

 Instruções para entrar na lista, sair da lista e usar a lista em
 http://www.mat.puc-rio.br/~obmlistas/obm-l.html
 
=




Re: [obm-l] MQ=MA=MG=MH

2012-06-16 Por tôpico Carlos Nehab

Eu já tinha notado o erro.
Abraços,
Nehab

Em 15/06/2012 19:05, Bernardo Freitas Paulo da Costa escreveu:

2012/6/15 Carlos Nehabcarlos.ne...@gmail.com:

Oi, Felippe,

Se o seu enunciado é:
Dentre os ternos (x, y, z) , com x, y e z reais, que satisfazem a x+y+z=5 e
xy+yz+xz=3 calcule o maior valor possível para x,
então eu achei outro resultado:
Usando (x+y+z)^2 = x^2+y^2+z^2 + 2(xy+yz+zx) obtemos
x^2+y^2+z^2= 19
Logo, o maior valor de x é raiz(19) que é maior que o 13/3 (e os
correspondentes valores de y e z são 0).

Cuidado, Nehab, porque x=raiz(19), y=z=0 não satisfaz x+y+z=5 (que é
racional) e, pior ainda, xy + yz + zx = 0, e não 3. Isso (mais uma
vez) serve como cota superior, mas não garante a existência da
solução.

Se der tempo (duvido...) eu mando uma na força bruta por
multiplicadores de Lagrange, que é sem dúvida mais geral do que a do
Ralph, e pode dar uma iluminada. (aliás, esse problema parece bastante
com outro dessa semana...)

abraços,


=
Instruções para entrar na lista, sair da lista e usar a lista em
http://www.mat.puc-rio.br/~obmlistas/obm-l.html
=


Re: [obm-l] MQ=MA=MG=MH

2012-06-15 Por tôpico Carlos Nehab

Oi, Felippe,

Se o seu enunciado é:
Dentre os ternos (x, y, z) , com x, y e z reais, que satisfazem a 
x+y+z=5 e xy+yz+xz=3 calcule o maior valor possível para x,

então eu achei outro resultado:
Usando (x+y+z)^2 = x^2+y^2+z^2 + 2(xy+yz+zx) obtemos
x^2+y^2+z^2= 19
Logo, o maior valor de x é raiz(19) que é maior que o 13/3 (e os 
correspondentes valores de y e z são 0).


Abraços
Nehab

Em 15/06/2012 13:50, Felippe Coulbert Balbi escreveu:

Olá a todos.
Eu queria saber condições para podermos usar qualquer uma dessas 
relações de desigualdades de forma que os termos sejam reais. Por exemplo:


x+y+z=5, xy+xz+yz=3 , qual o maior valor de x, para x,y,z E R?

yz= 3-x(5-x)
Usando MA= MG temos

5/3 = (x(3-x(5-x)) )^(1/3)

Resolvendo tal equação, chegamos em um número feio... porem, isso é 
falso ! o maior valor para x,y e z E R, é 13/3


Eu queria saber em que condições tal relação se torna valida.

Grato.
Coulbert




Re: [obm-l] Imagem

2012-06-11 Por tôpico Carlos Nehab

Oi, Ruy,

No caso geral de quociente entre duas expressões do segundo grau em x, é 
usual e simples chamar a fração de y e montar uma equação do segundo 
grau em x (que dependerá de y, naturalmente).

A pergunta é: para quais valores de y há x real?
Basta fazer brincar com o manjado delta.
É isto que você perguntou?
Porém, há uma solução mais fácil para este caso específico.
Sua f(x) = 1 - 2x/(x^2+x+1) = 1 - 2/(x + 1/x +1), para x  0.
Como |x + 1/x| =2 ...

Abraços,
Nehab

Em 11/06/2012 13:49, ruy de oliveira souza escreveu:
No exercício que pede o conjunto imagem da funcão real 
f(x)=(x^2-x+1)/(x^2+x+1) procedi da seguinte maneira: Fiz os gráficos 
das funções h(x)=x^2-x+1 e t(x)=x^2+x+1 e num mesmo sistema de 
coordenadas conclui que para -1=x=1 implica 1/3=f(x)=3. Para x=-1 
ou x=1, a imagem varia da mesma forma. A pergunta é: Existe um método 
não gráfico para determinarmos essa imagem?. Já aprendi aqui a 
resolução de problemas sobre imagens de funções fazendo-se uso da 
teoria de desigualdade de médias, mas acho que não se aplica a esse 
caso. Se houver um método não gráfico, com argumentos do ensino médio 
e alguém souber, agradeço antecipadamente. Abraços. 


=
Instruções para entrar na lista, sair da lista e usar a lista em
http://www.mat.puc-rio.br/~obmlistas/obm-l.html
=


Re: [obm-l] Fibonacci

2012-04-25 Por tôpico Carlos Nehab

Oi Marcone,

A forma mais simples de provar esta joça é usar duas coisinhas:

1) a^(km) - b^(km) é divisível por a^m - b^m   e
2) a formuleta do Binet para o termo geral da sequência de Fibonacci...

Tente! Na verdade esta estratégia mata zilhões de propriedades 
envolvendo Fibonacci de forma muito, mas muito simples.


Abraços
Nehab

Em 25/04/2012 09:21, marcone augusto araújo borges escreveu:

Prove que F_km é divisível por F_m(use indução em k)
Agradeço a quem puder ajudar.




Re: [obm-l] Soma

2012-04-23 Por tôpico Carlos Nehab

Oi, Smolka,

Na expressão do X - 2X  você se distraiu no sinal do  n.2^n que é menos.

Abraços
Nehab

Em 23/04/2012 16:45, J. R. Smolka escreveu:

Vejamos...

X = 1.2^0 + 2.2^1 + 3.2^2 + ... + n.2^(n - 1)
2X = 1.2^1 + 2.2^2 + 3.2^3 + ... + n.2^n

X - 2X = 1 + (2 - 1).2^1 + (3 - 2).2^2 + ... + [(n - 1) - (n - 
2)].2^(n - 1) + n.2^n

-X = 1 + 2 + 2^2 + 2^3 + ... + 2^(n - 1) + n.2^n

Os n primeiros termos do lado direito da equação formam uma PG com 
termo inicial a1 = 1 e razão r = 2. A soma destes n primeiros termos 
da PG é igual a:


Sn = a1.(1 - r^n) / (1 - r) = 1 - 2^n

então:

-X = 1 - 2^n + n.2^n = 1 - (n - 1).2^n == X = (n - 1).2^n - 1

Onde errei, então?

[ ]'s

*J. R. Smolka*

/Em 23/04/2012 13:15, Eduardo Wilner escreveu:/

Quase Smolka,

(n-1)2ˆn +1 .

[ ]`s





Re: [obm-l]

2012-04-21 Por tôpico Carlos Nehab

Oi, Ruy,

Acho que você vai gostar do texto
http://pt.scribd.com/doc/26528098/Medias-e-Desigualdades-Contatos-Imediatos-do-1%C2%BA-Grau-v5

Abraços
Nehab


Em 21/04/2012 12:15, ruy de oliveira souza escreveu:


Como se demonstra que para x=0 teremos x+1/x=2  sem o uso de 
limites? Quero dizer, uma provinha algébrica mesmo, sem uso de 
gráficos? Quem souber, agradeço antecipadamente. Abraços




=
Instruções para entrar na lista, sair da lista e usar a lista em
http://www.mat.puc-rio.br/~obmlistas/obm-l.html
=


Re: [obm-l] Re: [obm-l] aritmética

2012-04-16 Por tôpico Carlos Nehab

Caramba, Bernardo!

Você tem toda razão...  Obrigado pela correção!
De fato, então, talvez o único eventual mérito tenha sido obter as equações
(r1-1)(X+Y) = 10 e
(r2-1)(X+Y) = 7,
mais diretamente.
Daí, segue-se a solução dos colegas...,
Mais uma vez obrigado!
Abraços,
Nehab

Em 16/04/2012 03:20, Bernardo Freitas Paulo da Costa escreveu:

2012/4/16 Carlos Nehabcarlos.ne...@gmail.com:

2) Solução
X = 0,3737...  Y = 0,7373...
Na primeira base r1:
(r1^2-1).X = 3r1+7
(r1^2-1).Y = 7r1+3
Somando, (r1^2-1)(X+Y) = 10(r1+1), ou seja,
(r1-1)(X+Y)=10(A)
Dai já sabemos que r1-1 = 1, 2, 5 ou 10. Mas r1  7, logo r1 = 11 e X + Y =
1

Hum, X e Y são frações, certo? Porque então você insiste que X+Y seja
inteiro (única razão que eu vi para que r1 - 1 divida 10) ? Nesse caso
até que dá certo, mas sei lá, podia ser que X+Y = 1/2 (mas teria
talvez que mudar a expressão na base r2)

Abraços,


=
Instruções para entrar na lista, sair da lista e usar a lista em
http://www.mat.puc-rio.br/~obmlistas/obm-l.html
=


Re: [obm-l] aritmética

2012-04-15 Por tôpico Carlos Nehab

Oi, Jefferson,

Eu faria assim...

1) Explicação preliminar:
Se A = 0,4545...  (1)
é uma dízima na base 10,
usualmente sugiro aos alunos para ajeitar as coisas, multiplicando, 
neste caso, X por 100...

100.A = 45,4545...  (2)
Subtraindo (2) - (1) obtemos
99X = 45 = 4.10+5,
Em outra base b o que fozemos éequivalente a:
(b^2-1).X = 4b+5

2) Solução
X = 0,3737...  Y = 0,7373...
Na primeira base r1:
(r1^2-1).X = 3r1+7
(r1^2-1).Y = 7r1+3
Somando, (r1^2-1)(X+Y) = 10(r1+1), ou seja,
(r1-1)(X+Y)=10(A)
Dai já sabemos que r1-1 = 1, 2, 5 ou 10. Mas r1  7, logo r1 = 11 e X + 
Y = 1

(nem precisamos dos dados da outra base)
Usando os dados na outra base r2 obteríamos
(r2-1)(X+Y) = 7(B)
o que nos leva a r2 -1 =1 ou 7, mas r2 6  logo r2 = 8 e X+Y =1

Fazendo as continhas podemos obter X e Y que, valem X = 1/3 e Y = 2/3

Abraços
Nehab


Em 15/04/2012 18:39, Jefferson Franca escreveu:
Um aluno muito curioso e estudioso(tomara!) me deu esta questão 
durante uma aula semana passada e tentei, tentei e nada!

Será que alguém pode dar um ajuda aí?
Em uma base R1 uma fração F1se escreve como 0,373737...enquanto que 
uma fração F2é escrita como0,737373 . Em outra base R2, a fração F1é 
escrita como 0,252525... e a fração F2como 0,525252...A soma R1 + R2no 
sistema de numeração decimal é:

a) 24b) 22c) 21d) 20e) 19




Re: [obm-l] aritmética

2012-04-15 Por tôpico Carlos Nehab

Jefferson, apenas uma obs complementar:
Eu pensei que o problema desejava X+Y e não r1+r2. Logo a informação da 
segunda base é, obviamente, essencial, e não desnecessária como eu sugeri.

Abraços,
Nehab


Em 15/04/2012 23:17, Carlos Nehab escreveu:

Oi, Jefferson,

Eu faria assim...

1) Explicação preliminar:
Se A = 0,4545...  (1)
é uma dízima na base 10,
usualmente sugiro aos alunos para ajeitar as coisas, multiplicando, 
neste caso, X por 100...

100.A = 45,4545...  (2)
Subtraindo (2) - (1) obtemos
99X = 45 = 4.10+5,
Em outra base b o que fozemos éequivalente a:
(b^2-1).X = 4b+5

2) Solução
X = 0,3737...  Y = 0,7373...
Na primeira base r1:
(r1^2-1).X = 3r1+7
(r1^2-1).Y = 7r1+3
Somando, (r1^2-1)(X+Y) = 10(r1+1), ou seja,
(r1-1)(X+Y)=10(A)
Dai já sabemos que r1-1 = 1, 2, 5 ou 10. Mas r1  7, logo r1 = 11 e X 
+ Y = 1

(nem precisamos dos dados da outra base)
Usando os dados na outra base r2 obteríamos
(r2-1)(X+Y) = 7(B)
o que nos leva a r2 -1 =1 ou 7, mas r2 6  logo r2 = 8 e X+Y =1

Fazendo as continhas podemos obter X e Y que, valem X = 1/3 e Y = 2/3

Abraços
Nehab


Em 15/04/2012 18:39, Jefferson Franca escreveu:
Um aluno muito curioso e estudioso(tomara!) me deu esta questão 
durante uma aula semana passada e tentei, tentei e nada!

Será que alguém pode dar um ajuda aí?
Em uma base R1 uma fração F1se escreve como 0,373737...enquanto que 
uma fração F2é escrita como0,737373 . Em outra base R2, a fração F1é 
escrita como 0,252525... e a fração F2como 0,525252...A soma R1 + 
R2no sistema de numeração decimal é:

a) 24b) 22c) 21d) 20e) 19






Re: [obm-l] Re: [obm-l] Volume da pirâmide

2012-04-09 Por tôpico Carlos Nehab

Oi, Pedro Angelo,

Revendo as mensagens deste mês com mais tempo nestes feriados revi a sua.
Ai vai a dica clássica em belo e riquíssimo site:

http://myweb.lsbu.ac.uk/~whittyr/MathSci/TheoremOfTheDay/GeometryAndTrigonometry/EuclidsPrism/TotDEuclidsPrism.pdf

Você verá que o site http://www.theoremoftheday.org/; é uma bela 
referência.


Abraços
Nehab

Em 02/04/2012 22:16, Pedro Angelo escreveu:

Oi,
eu tentei bastante, mas não consegui mostrar que o volume do tetraedroé um 
terço do volume do prisma. Eu consigo dividir o prisma em trêstetraedros, sendo 
que dois deles são idênticos (cada um com uma dasbases do prisma), mas o 
terceiro tetraedro fica sempre diferente dosoutros, e aí eu não consigo mostrar 
que o volume dos três é igual.
2012/3/28 Paulo Césarpcesa...@gmail.com:  Olá Pedro.  Uma forma mais elementar consiste em se calcular o volume de um tetraedro a partir de um prisma triangular. Em 
seguida, aplica-se o Princípio de Cavalieri para uma pirâmide de base qualquer porém de mesma altura que o tetraedro. Dessa forma, conclui-se que o volume é de fato um terço do produto da 
área da base pela altura.  Att.  Paulo Cesar Sampaio Jr.  Enviado via iPad  Em 27/03/2012, às 21:04, Pedro Angelopedro.fon...@gmail.com  
escreveu:  Olá,  Sei que é possível achar o volume de uma pirâmide usando cálculo  integral, mas eu queria saber se há alguma demonstração mais  
elementar, como dizer que o triângulo tem a área igual a metade da  área do paralelogramo, que é base vezes altura. Eu podia jurar que eu  vi, um dia, o nosso amigo Nehab 
desenhar uma figurinha um tanto  elucidativa mas não consigo lembrar de jeito nenhum como era!  um abraço,  Pedro Ang!

elo  !

=  Instruções para entrar na lista, sair 
da lista e usar a lista em  http://www.mat.puc-rio.br/~obmlistas/obm-l.html  
=  
=  Instru�ões para entrar na lista, 
sair da lista e usar a lista em  http://www.mat.puc-rio.br/~obmlistas/obm-l.html  
=
=
Instru��es para entrar na lista, sair da lista e usar a lista em
http://www.mat.puc-rio.br/~obmlistas/obm-l.html
=




=
Instru��es para entrar na lista, sair da lista e usar a lista em
http://www.mat.puc-rio.br/~obmlistas/obm-l.html
=


Re: [obm-l] Enfado criativo... OFF TOPIC

2012-04-05 Por tôpico Carlos Nehab

Saudades, Marcelo
Grande abraço,
Nehab

Em 04/04/2012 22:01, Marcelo Salhab Brogliato escreveu:

Olá, Nehab, quanto tempo!!

Bom, vou tentar.. mas estou sem muitas idéias! =]

Python:
 len(set([ i*j for i in range(1, 21) for j in range(1, 21) if i != 
j ]))

139
Rsrs.. brincadeira! E não me precisa me sacanear, pra 10! ficará 
bastante lento, rs =]


Seja A_k = { 1k, 2k, ..., (k-1)k, (k+1)k, ..., 20k }.
Você quer saber |U_{k=1..20} A_k| = 380 - intersecções LOUCURA! 
hehehe =]
Ok, ok.. vou pra terceira tentativa.. (escrever enquanto pensa é bom.. 
fica o histórico)


Olhando para os números, só temos o fator primo 5 em: 5, 10, 15, 20.. 
e em todos esses casos ele tem expoente 1. Isto é, o maior expoente 
que podemos ter é 2. Logo, 5^3=125 está fora da contagem. Assim como, 
2*5^3 = 250, e 3*5^3 = 375. Logo, pensar somente nos primos, não 
resolve o problema por completo. Mas quanto nós erramos?


Bom, o maior valor sempre será (n-1)*n.. neste caso, 19*20 = 380.
Seja P = { x | 21 = x = 380 e x é primo }. É fácil ver que o produto 
de quaisquer primos em P sempre será maior que 380. Então, temos que 
tirar apenas os seus múltiplos. Mas quantos múltiplos temos de cada 
primo? Simples, [380/p_i] múltiplos. Assim, ficamos com: 380 - sum{p_i 
\in P} [380/p_i], onde [x] é o piso de x.
Fazendo esta conta, ficamos com 197... conforme esperado, é maior que 
a resposta correta, 139.


Próxima tentativa.. :)

Ainda tem os chatos que se repetem. Vejamos: (2*3)*(3*2*2) = 
(2*2)*(3*3*2)... isto é: 6*12 = 4*18... esses são os chatos que estão 
me atrapalhando a vida.. rs! Outro chato é: (2*2)*(2*2*2) = 
2*(2*2*2*2), isto é: 2 * 16 = 4 * 8.. ah, se eu conseguisse contá-los..


Bom, vou tentar mais depois e eu envio..
Espero que essa confusão de idéias possa ajudar alguém a resolver o 
problema, hehe.


Abração,
Salhab






2012/4/3 Carlos Nehab carlos.ne...@gmail.com 
mailto:carlos.ne...@gmail.com


Oi, colegas,

Enfadado, fui fazer o que professor gosta: inventar moda para
enfernizar a vida dos alunos (no bom sentido, é claro...).
É um mesmo exercício em várias versões.
Divirtam-se.

Versão 1:
Dado o conjunto A { 1, 2, 3,..., 20}, escolha quaisquer dois
elementos distintos deste conjunto e multiplique-os.
Se você fizer isto para todas as situações possíveis, respeitando
o fato de que os números escolhidos não podem ser iguais,  quantos
resultados diferentes você obterá?

Versão 2:
Idem com o conjunto dos inteiros de  1 a 10! (fatorial de 10).

Versão 3:
Idem com o conjunto A = { 1, 2, 3, ..., n}, n  1.

Abraços
Nehab

=
Instruções para entrar na lista, sair da lista e usar a lista em
http://www.mat.puc-rio.br/~obmlistas/obm-l.html
http://www.mat.puc-rio.br/%7Eobmlistas/obm-l.html
=






Re: [obm-l] Enfado criativo...

2012-04-04 Por tôpico Carlos Nehab

Oi, José Carneiro,

Não está correto não.
Desejamos a qde de resultados *diferentes* e não a quantidade de 
produtos possíveis ou similar.


Abraços
Nehab

Em 03/04/2012 10:57, JOSE AIRTON CARNEIRO escreveu:

(An,2)/2.
Em 3 de abril de 2012 00:43, Carlos Nehab carlos.ne...@gmail.com 
mailto:carlos.ne...@gmail.com escreveu:


Oi, colegas,

Enfadado, fui fazer o que professor gosta: inventar moda para
enfernizar a vida dos alunos (no bom sentido, é claro...).
É um mesmo exercício em várias versões.
Divirtam-se.

Versão 1:
Dado o conjunto A { 1, 2, 3,..., 20}, escolha quaisquer dois
elementos distintos deste conjunto e multiplique-os.
Se você fizer isto para todas as situações possíveis, respeitando
o fato de que os números escolhidos não podem ser iguais,  quantos
resultados diferentes você obterá?

Versão 2:
Idem com o conjunto dos inteiros de  1 a 10! (fatorial de 10).

Versão 3:
Idem com o conjunto A = { 1, 2, 3, ..., n}, n  1.

Abraços
Nehab

=
Instruções para entrar na lista, sair da lista e usar a lista em
http://www.mat.puc-rio.br/~obmlistas/obm-l.html
http://www.mat.puc-rio.br/%7Eobmlistas/obm-l.html
=






[obm-l] Enfado criativo...

2012-04-02 Por tôpico Carlos Nehab

Oi, colegas,

Enfadado, fui fazer o que professor gosta: inventar moda para enfernizar 
a vida dos alunos (no bom sentido, é claro...).

É um mesmo exercício em várias versões.
Divirtam-se.

Versão 1:
Dado o conjunto A { 1, 2, 3,..., 20}, escolha quaisquer dois elementos 
distintos deste conjunto e multiplique-os.
Se você fizer isto para todas as situações possíveis, respeitando o fato 
de que os números escolhidos não podem ser iguais,  quantos resultados 
diferentes você obterá?


Versão 2:
Idem com o conjunto dos inteiros de  1 a 10! (fatorial de 10).

Versão 3:
Idem com o conjunto A = { 1, 2, 3, ..., n}, n  1.

Abraços
Nehab

=
Instruções para entrar na lista, sair da lista e usar a lista em
http://www.mat.puc-rio.br/~obmlistas/obm-l.html
=


[obm-l] Re: [obm-l] Re: [obm-l] Re: [obm-l] Re: [obm-l] Re: [obm-l] heptágono regular

2012-03-28 Por tôpico Carlos Nehab

Perfeito Pedro.
Ótimo comentário considerando a garotada de 1a e 2a série!
E as respostas da múltipla escolha ajudam...
Confesso que não reparei que a circunferência era menor que 16...
Abraços
Nehab

Em 28/03/2012 15:24, Pedro José escreveu:

Desculpe-me pela  intromissão. Porém, a depender da série, nem sempre
temos disponível o valor da função seno para os arcos que não sejam
notáveis.
Portanto é melhor ordená-lo entre o perímetro do hexágono e o
comprimento da circunferência circunscrita  ao pentágono.
L = 2πR = 2*2,5*π = 5π  16.
Creio que se fosse para calcular com o sen(22,5 graus), melhor seria
calcular direto com o sen(32 graus). Utilizando a circunferência como
parâmetro - para esse caso em si - se evita a necessidade de
conhecimento ou uso de tabela para obter o valor da função sen.

Saúde, paz e alegria!


Em 26/03/12, felipe araujo costafaraujoco...@yahoo.com.br  escreveu:

Olá Carlos.
Esse exercicio foi um aluno que pediu a questao foi do Colegio Naval por
isso queria saber uma soluçao por desigualdade de triangulos.
Muito Obrigado.

Felipe Araujo Costa
Cel: 77430066
E-mail: faraujoco...@yahoo.com.br
faco...@metalmat.ufrj.br



  De: Carlos Nehabcarlos.ne...@gmail.com
Para: obm-l@mat.puc-rio.br
Enviadas: Segunda-feira, 26 de Março de 2012 10:26
Assunto: Re: [obm-l] Re: [obm-l] Re: [obm-l] heptágono regular


Oi, Felipe,
Sim, Felipe, há uma solução por desigualdades, que funciona porque
 as opções facilitam...
Solução banal:  analisando o perímetro do hexagono e do octógono
 regular inscritos no mesmo círculo (o perímetro do heptágono estará
 entre eles...)
Perímetro do hexagono = 6x2,5 = 15 (logo, a opção a está errada)
Perímetro do octógono é 8x2x2,5xsen22,5=40sen22,5
Mas sen^2(22,5) = (1-cos45)/2=0,15. Como raiz(0,15)0,4, o
 perímetro do octógono16
Resposta: opção b - entre 15 e 16.
Pronto. Este exercício foi proposto para qual série?

Abraços,
Nehab


Em 25/03/2012 11:21, felipe araujo costa escreveu:
Obrigado Érica.

Mas queria saber se ha uma soluçao por desigualdade dos lados.
Abraço.

Felipe Araujo Costa
Cel: 77430066
E-mail: faraujoco...@yahoo.com.br
faco...@metalmat.ufrj.br



De: Érica Gualberto Pongelupe Giacoiaprofer...@ig.com.br
Para: obm-l@mat.puc-rio.br
Enviadas: Domingo, 25 de Março de 2012 10:30
Assunto: [obm-l] Re: [obm-l] heptágono regular


Use a lei dos cossenos e calcule a medida do lado x.
x^2=2,5^2+2,5^2-2*2,5*2,5*cos(360/7)
Depois, basta multiplicar x por 7.
Abração


Em 25 de março de 2012 10:14, felipe araujo costa
faraujoco...@yahoo.com.br  escreveu:

Bom dia.

Preciso de uma ajuda nessa questão. Quero saber se ha uma resolução por
desigualdade entre os lados do heptagono.
Obrigado.




* O perímetro do heptágono regular convexo inscrito num círculo de raio
2,5, é um número  real que esta entr


a)14 e 15
b)15 e 16
c)16 e 17
d)17 e 18
e)18 e 19

Felipe Araujo Costa




--
Érica G. P. G.




=
Instru��es para entrar na lista, sair da lista e usar a lista em
http://www.mat.puc-rio.br/~obmlistas/obm-l.html
=


=
Instru��es para entrar na lista, sair da lista e usar a lista em
http://www.mat.puc-rio.br/~obmlistas/obm-l.html
=


[obm-l] Re: [obm-l] Re: [obm-l] Re: [obm-l] Re: [obm-l] Re: [obm-l] Re: [obm-l] Re: [obm-l] Re: [obm-l] heptágono regular

2012-03-28 Por tôpico Carlos Nehab

Bouskela e Bernardo,

Sem entrar no mérito do argumento como um todo, o item 5 pode ser 
desenvolvido de forma mais simples, não exigindo a elipse:

Sendo O é o centro do circulo´e os vértices consecutivos A, B e C, façamos:
AOC = alfa (fixo),
AOB = beta e
BOC = gama,
O comprimento AB + BC é dado por
2Rsen(beta/2) + 2Rsen(gama/2) = 4Rsen(alfa/4).cos(beta/4-gama/4) que é 
máximo quando cos = 1 ou seja, beta = gama.


Quanto à preocupação do Bernardo com a justificável precisão de 
convergência, etc, quem trabalha com meninos do nivel médio (como eu, em 
turmas de preparação ao IME/ITA) tem dúzias de preocupações 
assemelhadas e até em assuntos aparentemente óbvios.
Por exemplo: prove que a área de um retângulo é igual ao produto dos 
lados. Acha fácil?
Não é não, pois se os lados do retângulo são múltiplos racionais do lado 
do quadrado que serve de unidade de medida, tudo bem, mas se não forem, 
olha a confusão!
Tem que falar em aproximação de irracionais por racionais ou coisa 
análoga ou ficar de boca fechada e tocar o bonde ou, se a turma for 
melhorzinha (tipo IME/ITA, etc) ai tá, damos uma pincelada na questão...
E na primeira série?  Soma das PGs ilimitadas são apresentadas sem 
nenhuma formalização do conceito de limite, etc e pronto;
E dízimas para as pobres criancinhas... tantos noves ... tantos zeros 
quanto ! Agh.  Sempre achei equivalente a assassinato o 
estudo de dízimas na época em que é feito.

E por ai vai.

Abraços
Nehab

PS: Acho que me excedi... e me empolguei. Faz parte.


Em 28/03/2012 18:09, Bernardo Freitas Paulo da Costa escreveu:

2012/3/28 Albert Bouskelaalb...@themag.com.br:
Ois!

Antes de mais nada, parabéns ao Albert por ter matado o problema.
Enfim, a sacada de gênio, claro, porque se eu estou escrevendo um
mail a mais, é que eu ainda não estou satisfeito... mas é pura chatice
minha. Coisas da idade.


 3)Mantendo-se invariante o número de vértices, o polígono
inscrito num círculo que tem (refiro-me ao polígono) o maior perímetro é o
polígono regular!

 4)Prova: – Peguem 3 pontos consecutivos (adjacentes) de um
polígono. O 1º e o 3º formam a corda que nos interessa. Os 3 pontos formam
um triângulo. Para que o perímetro desse triângulo seja máximo, ele deve ser
isósceles (prova adiante). I.e., o 2º ponto deve coincidir com a interseção
da maior flecha da corda formada pelos 1º e 3º pontos – i.e., a mediatriz da
corda – com o círculo. Por quê?

Eu não diria que é o perímetro do triângulo, mas apenas AB + BC (onde
A, B e C são, nessa ordem, os pontos consecutivos). Claro que como
apenas B mexe, somar AC é uma constante, logo não muda nada. Mas o
importante ainda está por vir.


 5)Porque quando o ponto do meio (o 2º) passeia ao longo do
arco compreendido pela corda, a partir do ponto central (o ponto de
interseção da mediatriz da corda com o círculo), o perímetro do triângulo
diminui! Caso não diminuísse (ficasse constante),

Na minha terra, uma quantidade que não diminui, pode aumentar.


o LG do 2º ponto seria uma
elipse, com polos no 1º e 3º pontos – lembrem-se da definição da elipse (é o
LG dos pontos cuja soma das distâncias do ponto considerado até dois pontos
fixos, os polos, é invariante). Esta elipse passaria “por fora” do círculo,
tangenciando-o no ponto de interseção da mediatriz da corda com o círculo.

Mas o argumento da elipse ainda assim é válido. Mas tem que dar uns
detalhes a mais: considere a elipse de focos A e C passando por B.
Como B está na mediatriz de AC, a elipse é tangente ao círculo em B.
As posições de uma elipse e um círculo são bem simples: é uma questão
de curvatura. Ora, a elipse com certeza passa fora do círculo na
altura da reta AC, logo ela também tem que estar do lado de fora
acima, logo a semi-elipse A'BC' contém o arco ABC. Como os pontos
dentro da elipse são os que têm a soma MENOR do que AB+BC, acabou.


 6)Já que o triângulo deve ser isósceles, então o polígono
deve ser regular!

Muita calma nessa hora. Seja P um polígono que não é regular. Seja P'
o polígono obtido de P isoscelizando dois lados contíguos como o
Albert fez. Então per(P)  per(P'). Em particular, se P for regular, o
procedimento do Albert não permite aumentar o comprimento, mas é só
isso. Claro que se você tomar outro polígono (que não seja regular,
portanto) ele não será o de maior perímetro, mas isso não implica que
o regular seja o maximizador.

Assim, aqui vem a minha chatice maior. Falta um de dois argumentos. Um
de convergência (se a gente tiver sorte, para o máximo), o outro de
existência do dito máximo.

A existência é um treco chato, e eu não sei provar sem
análise/topologia. O máximo existe porque o comprimento de um polígono
é uma função contínua dos ângulos dos vértices do mesmo, que vivem num
conjunto compacto (o círculo) e são em número finito. Sabendo da
existência, acabou: seja P o polígono que é de maior perímetro. A
construção do Bouskela diz que P tem que ser regular, já que, se 

[obm-l] Re: [obm-l] Re: [obm-l] Re: [obm-l] Re: [obm-l] heptágono regular

2012-03-27 Por tôpico Carlos Nehab

Caros todos amigos,

Eu tinha certeza que iria haver polêmica.
Quem acompanhou todas as discussões sobre esta questão, pode verificar 
que eu, inicialmente, nem havia reparado que o heptágono era regular e 
viajei em temas muito mais complicados supondo a questão quente.
Quando caiu a ficha que era regular, usei apenas o bom senso de uma 
solução intuitiva e adequada para o ensino médio convencional.
De qualquer forma, procedem as reclamações sobre o não é tão óbvio 
assim, mas não esqueçam que a questão era de múltipla escolha...


Como bem disse o Bernardo, ficamos todos devendo uma explicação banal 
(no sentido do nivel médio convencional) para o fato de que o perímetro 
é crescente com o número de lados.


Abraços
Nehab


Em 26/03/2012 22:41, felipe araujo costa escreveu:

Olá Carlos.
Esse exercicio foi um aluno que pediu a questao foi do Colegio Naval 
por isso queria saber uma soluçao por desigualdade de triangulos.

Muito Obrigado.
Felipe Araujo Costa
Cel: 77430066
E-mail: faraujoco...@yahoo.com.br
faco...@metalmat.ufrj.br

*De:* Carlos Nehab carlos.ne...@gmail.com
*Para:* obm-l@mat.puc-rio.br
*Enviadas:* Segunda-feira, 26 de Março de 2012 10:26
*Assunto:* Re: [obm-l] Re: [obm-l] Re: [obm-l] heptágono regular

Oi, Felipe,
Sim, Felipe, há uma solução por desigualdades, que funciona porque as 
opções facilitam...
Solução banal:  analisando o perímetro do hexagono e do octógono 
regular inscritos no mesmo círculo (o perímetro do heptágono estará 
entre eles...)

Perímetro do hexagono = 6x2,5 = 15 (logo, a opção _a_ está errada)
Perímetro do octógono é 8x2x2,5xsen22,5=40sen22,5
Mas sen^2(22,5) = (1-cos45)/2=0,15. Como raiz(0,15) 0,4, o perímetro 
do octógono 16

Resposta: opção b - entre 15 e 16.
Pronto. Este exercício foi proposto para qual série?

Abraços,
Nehab


Em 25/03/2012 11:21, felipe araujo costa escreveu:

Obrigado Érica.
Mas queria saber se ha uma soluçao por desigualdade dos lados.
Abraço.
Felipe Araujo Costa
Cel: 77430066
E-mail: faraujoco...@yahoo.com.br mailto:faraujoco...@yahoo.com.br
faco...@metalmat.ufrj.br mailto:faco...@metalmat.ufrj.br

*De:* Érica Gualberto Pongelupe Giacoia profer...@ig.com.br 
mailto:profer...@ig.com.br

*Para:* obm-l@mat.puc-rio.br mailto:obm-l@mat.puc-rio.br
*Enviadas:* Domingo, 25 de Março de 2012 10:30
*Assunto:* [obm-l] Re: [obm-l] heptágono regular

Use a lei dos cossenos e calcule a medida do lado x.
x^2=2,5^2+2,5^2-2*2,5*2,5*cos(360/7)
Depois, basta multiplicar x por 7.
Abração

Em 25 de março de 2012 10:14, felipe araujo costa 
faraujoco...@yahoo.com.br mailto:faraujoco...@yahoo.com.br escreveu:


Bom dia.
Preciso de uma ajuda nessa questão. Quero saber se ha uma
resolução por desigualdade entre os lados do heptagono.
Obrigado.


  * O perímetro do heptágono regular convexo inscrito num círculo
de raio 2,5, é um número  real que esta entr

a)14 e 15
b)15 e 16
c)16 e 17
d)17 e 18
e)18 e 19

Felipe Araujo Costa




--
Érica G. P. G.










[obm-l] Re: [obm-l] Re: [obm-l] Re: [obm-l] Re: [obm-l] Re: [obm-l] Re: [obm-l] heptágono regular

2012-03-27 Por tôpico Carlos Nehab

Oi,Marcos,

Se desculpar! Nada, você tá certo e eu é que agradeço esta sua ótima 
contribuição. Gostei e o copirraite tá garantido.
Mas acho que conseguirei uma solução nivel médio convencional para o 
fato do perímetro ser crescente com o número de lados...

Abraços
Nehab

Em 27/03/2012 10:25, Marcos Martinelli escreveu:
Peço desculpas por ter sido muito formal nesta questão. É que, pra 
mim, realmente não foi tão intuitivo supor que o perímetro seria 
crescente. Deve haver sim uma solução por geometria pura, mas ficarei 
devendo.


Agora, quanto à questão levantada pelo último email, posso contribuir 
um pouco:


Suponha um polígono qualquer, de N lados, inscrito em uma 
circunferência de raio R. Vou tentar provar que seu perímetro é sempre 
menor ou igual ao perímetro do polígono regular de N lados inscrito 
nessa mesma circunferência.


Seja 2p_(N) seu perímetro, que poderá ser calculado da seguinte maneira:

2p_(N) = sum_{i=1}^{n} 2R . sen (teta_i / 2), onde teta_i é o ângulo 
formado pelo centro do círulo (O) e os vértices P_i e P_i + 1 (sendo o 
índice i visto como sendo tomado módulo N. Ou seja, P_(N + 1) 
corresponde a P_1).


Importante notar que aqui a prova se subdivide em duas possibilidades:

i) O está contido no polígono. Repare que isso implica sum_{i=1}^{N} 
teta_i = 2 . pi (*)


Voltando ao perímetro exposto acima, teríamos:

2p_(N) =  2R . sum_{i=1}^{n} sen (teta_i / 2) = 2RN . [(sum_{i=1}^{n} 
sen (teta_i / 2)) / N]. Aqui vamos aplicar a desigualdade de Jensen 
(para isso, observe que 0 = teta_i/2 = pi / 2).


Vamos considerar, então, a função: f(0,pi/2) -- R tal que f(x) = 
sen(x/2). Repare que f´´(x) = - 1/4 . sen(x/2)  0 para todo x no 
domínio (0,pi/2). Assim, podemos afirmar:


(sum_{i=1}^{n} sen (teta_i / 2)) / N = sen(1/2 . (sum_{i=1}^{N} 
teta_i) / N) = sen( pi / N). Essa última igualdade ocorre por causa de 
(*).


Portanto: 2p_(N) = 2RN . sen(pi/N). Aqui a igualdade só ocorre se 
todos os teta_i forem iguais, o que implica ser o polígono regular.


ii) O não está contido no polígono. Perdemos, então, a igualdade (*). 
Mas ainda temos que vale Jensen. Portanto:


2p_(N) =  2R . sum_{i=1}^{n} sen (teta_i / 2) = 2RN . [(sum_{i=1}^{n} 
sen (teta_i / 2)) / N] = 2RN . sen(1/2 . (sum_{i=1}^{N} teta_i) / N).


Aqui temos uma particularidade interessante: um dos ângulos teta_i, 
digamos o N-ésimo (SPG), é igual a soma de todos os demais. 
Assim: 2p_(N) = 2RN . sen(teta_N / N)  2RN . sen(pi / N). Essa 
última desigualdade vale porque teta_N é sempre inferior a pi.


=
Instruções para entrar na lista, sair da lista e usar a lista em
http://www.mat.puc-rio.br/~obmlistas/obm-l.html
=


Re: [obm-l] Re: [obm-l] Re: [obm-l] heptágono regular

2012-03-26 Por tôpico Carlos Nehab

Oi, Felipe,
Sim, Felipe, há uma solução por desigualdades, que funciona porque as 
opções facilitam...
Solução banal:  analisando o perímetro do hexagono e do octógono regular 
inscritos no mesmo círculo (o perímetro do heptágono estará entre eles...)

Perímetro do hexagono = 6x2,5 = 15 (logo, a opção _a_ está errada)
Perímetro do octógono é 8x2x2,5xsen22,5=40sen22,5
Mas sen^2(22,5) = (1-cos45)/2=0,15. Como raiz(0,15) 0,4, o perímetro do 
octógono 16

Resposta: opção b - entre 15 e 16.
Pronto. Este exercício foi proposto para qual série?

Abraços,
Nehab


Em 25/03/2012 11:21, felipe araujo costa escreveu:

Obrigado Érica.
Mas queria saber se ha uma soluçao por desigualdade dos lados.
Abraço.
Felipe Araujo Costa
Cel: 77430066
E-mail: faraujoco...@yahoo.com.br
faco...@metalmat.ufrj.br

*De:* Érica Gualberto Pongelupe Giacoia profer...@ig.com.br
*Para:* obm-l@mat.puc-rio.br
*Enviadas:* Domingo, 25 de Março de 2012 10:30
*Assunto:* [obm-l] Re: [obm-l] heptágono regular

Use a lei dos cossenos e calcule a medida do lado x.
x^2=2,5^2+2,5^2-2*2,5*2,5*cos(360/7)
Depois, basta multiplicar x por 7.
Abração

Em 25 de março de 2012 10:14, felipe araujo costa 
faraujoco...@yahoo.com.br mailto:faraujoco...@yahoo.com.br escreveu:


Bom dia.
Preciso de uma ajuda nessa questão. Quero saber se ha uma
resolução por desigualdade entre os lados do heptagono.
Obrigado.


  * O perímetro do heptágono regular convexo inscrito num círculo
de raio 2,5, é um número  real que esta entr

a)14 e 15
b)15 e 16
c)16 e 17
d)17 e 18
e)18 e 19

Felipe Araujo Costa




--
Érica G. P. G.






Re: [obm-l] geometria

2012-03-25 Por tôpico Carlos Nehab

Ora, ora,

E eu não li o enunciado direito e nem percebi que seu heptágono era regular!
Mais certamente seria BEM MAIS interessante se não fosse...
Abraços
Nehab


Em 23/03/2012 15:10, Carlos Nehab escreveu:

Oi, Felipe,

Bonito problema e confesso que não o conhecia e não saquei solução.
Mas descobri vários artigos sobre o tema (o que por si só denota que 
não deve se tratar de problema banal).


Veja em http://dl.acm.org/citation.cfm?id=1501726

A polygon is said to be /simple/ if the only points of the plane 
belonging to two of its edges are its vertices.
We answer the question of finding, for a given integer /n/, a simple 
/n/-sided polygon contained in a disk of radius 1 that has the longest 
perimeter.
When /n/ is even, the optimal solution is arbitrarily close to a line 
segment of length 2/n/. When /n/ is odd, the optimal solution is 
arbitrarily close to an isosceles triangle.

Ou em
https://springerlink3.metapress.com/content/271w6pw85j59x451/resource-secured/?target=fulltext.pdfsid=aoia4fxk1sfpnwuahjnrq21xsh=www.springerlink.com

Mas você gostará de ler o artigo em
http://www.gerad.ca/Charles.Audet/PUB/extremal.pdf

Adoraria que alguém mais esperto do que eu oferecesse uma solução 
simples para seu problema.


Abraços
Nehab


Em 22/03/2012 00:45, felipe araujo costa escreveu:

Preciso de um ajuda.

Qual intervalo que o perímetro de um heptágono regular assume estando 
inscrito numa circunferência de raio 2,5 cm?


Desde já agradeço.






Re: [obm-l] geometria

2012-03-23 Por tôpico Carlos Nehab

Oi, Felipe,

Bonito problema e confesso que não o conhecia e não saquei solução.
Mas descobri vários artigos sobre o tema (o que por si só denota que não 
deve se tratar de problema banal).


Veja em http://dl.acm.org/citation.cfm?id=1501726

A polygon is said to be /simple/ if the only points of the plane 
belonging to two of its edges are its vertices.
We answer the question of finding, for a given integer /n/, a simple 
/n/-sided polygon contained in a disk of radius 1 that has the longest 
perimeter.
When /n/ is even, the optimal solution is arbitrarily close to a line 
segment of length 2/n/. When /n/ is odd, the optimal solution is 
arbitrarily close to an isosceles triangle.

Ou em
https://springerlink3.metapress.com/content/271w6pw85j59x451/resource-secured/?target=fulltext.pdfsid=aoia4fxk1sfpnwuahjnrq21xsh=www.springerlink.com

Mas você gostará de ler o artigo em
http://www.gerad.ca/Charles.Audet/PUB/extremal.pdf

Adoraria que alguém mais esperto do que eu oferecesse uma solução 
simples para seu problema.


Abraços
Nehab


Em 22/03/2012 00:45, felipe araujo costa escreveu:

Preciso de um ajuda.

Qual intervalo que o perímetro de um heptágono regular assume estando 
inscrito numa circunferência de raio 2,5 cm?


Desde já agradeço.




Re: [obm-l] Raizes da equação

2012-03-09 Por tôpico Carlos Nehab

Oi, Felipe,

Equações do terceiro grau cujos coeficientes de x^3 e x  estão na razão 
4 para 3 chamam a atenção por conta de

cos3A = 4(cosA)^3 - 3.(cosA)

Sua equação esconde um pouco o 4 para 3 mas se você fizer x = 2cosA  
você cairá na equação óbvia cos3A = 1/2.


Abraços,
Nehab


Em 08/03/2012 17:28, felipe araujo costa escreveu:

Boa tarde.
Como achar as raizes da seguinte eq. x^3 - 3x - 1 = 0 ?
Felipe Araujo Costa




Re: [obm-l] (EN) Contagem

2012-02-24 Por tôpico Carlos Nehab

Gostei.
Bem mais simples que a minha sugestão.
Abraços
Nehab

Em 24/02/2012 00:33, terence thirteen escreveu:

Poxa, gente, é mais fácil que isso!
Todos os números que só tem 2,3,4,5,6? Simples: associe cada um delescom aquele 
que seja feito trocando os dígitos por aqueles que faltampra somar 8.
Enfim, troque 2 com 6, 3 com 5 e 4 com 4.
Exemplo: 24365--  64523. É impossível que dẽ dois caras iguais - sóse fosse 
tudo 4, mas só vale repetidos.
Assim, a soma desses pares é 8. Multiplica pelo tanto de pares e fim!
Como sempre, Gauss somando de 1 até 100 na raça...
Em 23 de fevereiro de 2012 22:06, Carlos Nehabcarlos.ne...@gmail.com  escreveu:  Poxa, Arkon,  No braço? Se poupe...  Imagine que você esteja somando todos o números na 
vertical. As unidades  primeiro: quantas vezes você somará algarismos 2, algarismos 3, etc?  Abraços,  Nehab  Em 23/02/2012 20:40, arkon 
escreveu:  Pessoal, qual o bizu ou é só no braço mesmo???  Com os algarismos 2, 3, 4, 5 e 6 formam-se todos os números de 5  algarismos 
distintos.  Determine a soma de todos eles.  =  Instru��es para entrar na lista, sair da lista e 
usar a lista em  http://www.mat.puc-rio.br/~obmlistas/obm-l.html  =  
=  Instru�ões para entrar na lista, sair da lista e usar a lista em  http://www.mat.puc-rio.br/~obmlistas/ob!

m-l.html  ===!

==


-- /**/神が祝福
Torres
=
Instru��es para entrar na lista, sair da lista e usar a lista em
http://www.mat.puc-rio.br/~obmlistas/obm-l.html
=




=
Instru��es para entrar na lista, sair da lista e usar a lista em
http://www.mat.puc-rio.br/~obmlistas/obm-l.html
=


Re: [obm-l] (EN) Contagem

2012-02-23 Por tôpico Carlos Nehab

Poxa, Arkon,
No braço? Se poupe...
Imagine que você esteja somando todos o números na vertical. As unidades 
primeiro: quantas vezes você somará algarismos 2, algarismos 3, etc?

Abraços,
Nehab


Em 23/02/2012 20:40, arkon escreveu:


Pessoal, qual o bizu ou é só no braço mesmo???

Com os algarismos 2, 3, 4, 5 e 6 formam-se todos os números de 5 
algarismos distintos.
Determine a soma de todos eles. 
= 
Instru��es para entrar na lista, sair da lista e usar a lista em 
http://www.mat.puc-rio.br/~obmlistas/obm-l.html 
= 


=
Instru��es para entrar na lista, sair da lista e usar a lista em
http://www.mat.puc-rio.br/~obmlistas/obm-l.html
=


Re: [obm-l] Ajuda

2012-02-20 Por tôpico Carlos Nehab

  
  
Oi, Julio,
  
  No resisti a dar uma dica, emborao utros colegas j tenham
  resolvido o problema... 
  Este tipo de exerccio, envolvendo circunferncias e retas so
  muito comuns e as pessoas tentam solues via Geometria Analtica,
  s vezes chatssimas.
  Entretanto, Geometria Analtica tambm  Geometria e uma
  figurinha (mais um pitaggoras) geralmente resolvem o problema
  facilmente...
  Veja, na figura o tringulo retngulo tem lados bvios (1, 4, 2
  2raiz2) e ento a tangente de alfa  imediata...Dai as inclinaes
  das duas retas so imediatas...


Abraos,
Nehab

Em 14/02/2012 16:33, Julio Teixeira escreveu:

  bom dia, estudando me deparei com este exercicio, onde encontrei certa
dificuldade e nao consegui resolve-lo, assim peco ajuda em como
prosseguir..


Determine a equao de todas as retas que so tangentes 
circunferncia x + y = 2y e passam pelo ponto (0,4).

Agradecido desde ja, aguardando retorno..

=
Instrues para entrar na lista, sair da lista e usar a lista em
http://www.mat.puc-rio.br/~obmlistas/obm-l.html
=



  



Re: [obm-l] off topic (livro de geometria)

2012-01-21 Por tôpico Carlos Nehab

Hahaha,

Caramba, estudei nele.
Só não vou dizer há quantas décadas! É ótimo!

Abraços,
Nehab

Em 19/01/2012 14:33, staib escreveu:
Boa tarde. Alguém saberia me dizer se o livro de geometria Irmãos 
Marista é realmente bom?

Abraços




Re: [obm-l] Área da elipse

2012-01-04 Por tôpico Carlos Nehab

  
  
Troquei a com b, mas acho que o esprito t claro.

On 04/01/2012 20:08, Carlos Nehab wrote:

  
  Bem, Joo,
  
  A soluo geomtrica  mais adequada e simples.
  Se voc sabe (ou pode usar) o fato de que a projeo de um crculo
  sobre um plano  uma elipse, ento h uma soluo "nivel mdio"
  trivial.
  Veja a figurinha:
  
  
  
  
  A rea do crculo  pi.a^2 e a rea da elipse  ela vezes o cos
  alfa, logo,  pi.a^2. (b/a) = pi.ab.
  
  Abraos
  Nehab
  
  
  On 04/01/2012 18:34, Joo Maldonado wrote:
  

Algum sabe alguma demonstrao fcil da rea da
  elipse sem usarintegral?
  Casono haja, algum sabede alguma que possa serr
  resolvida poralguma substituio (do jeit o que estou
  tentando fazer s apelando para o wolfram mesmo )
  
  []s
  Joo

  
  


  



Re: [obm-l] Área da elipse

2012-01-04 Por tôpico Carlos Nehab

  
  
Bem, Joo,

A soluo geomtrica  mais adequada e simples.
Se voc sabe (ou pode usar) o fato de que a projeo de um crculo
sobre um plano  uma elipse, ento h uma soluo "nivel mdio"
trivial.
Veja a figurinha:




A rea do crculo  pi.a^2 e a rea da elipse  ela vezes o cos
alfa, logo,  pi.a^2. (b/a) = pi.ab.

Abraos
Nehab


On 04/01/2012 18:34, Joo Maldonado wrote:

  
  Algum sabe alguma demonstrao fcil da rea da
elipse sem usarintegral?
Casono haja, algum sabede alguma que possa serr resolvida
poralguma substituio (do jeit o que estou tentando fazer s
apelando para o wolfram mesmo )

[]s
Joo
  


  



Re: [obm-l] Re: [obm-l] Re: [obm-l] Função Injetora

2011-12-14 Por tôpico Carlos Nehab

Caro Bernardo et alli,
Contrariando Goedel, como sempre, você continua_completo e consistente_ 
nas suas belas intervenções...

Abraços do admirador,
Nehab

On 13/12/2011 19:46, Bernardo Freitas Paulo da Costa wrote:

2011/12/13 Rodrigo Renjirodrigo.uff.m...@gmail.com:

Olá joão!

Isso não vale em geral em conjuntos infinitos

considere por exemplo

f: N em N com
f(n) =n+1

a função é injetora, porém não é sobrejetora.

nenhum elemento é enviado no número 0 ( com N= {0,1,2,3,} )

Só para completar: o exemplo do Renji e a questão do João são
universais. Ou seja, vale o seguinte:

- se um conjunto é finito, então toda função injetora é bijetora,
- se um conjunto é infinito, então existe uma função injetora que não
é bijetora.

Para ver a segunda parte, basta lembrar que cada conjunto infinito
contém uma cópia de N dentro dele, e daí você usa uma função que é a
função do Renji na cópia do N, e identidade no resto (se houver),
que é injetiva e não sobrejetiva.

Em uma frase só: um conjunto é finito se, e somente se, toda função
injetora é bijetora.

Ah, quase esqueci: um conjunto X é infinito quando ele não é finito
(daã), ou seja, quando não existir uma bijeção de {0, 1, ..., n}
em X. Se existisse uma sobrejeção de  {0, 1, ..., n} em X mas não
bijeção, X é finito também, porque os subconjuntos de um conjunto
finito são finitos, e X seria bijetivo com a um subconjunto de {0, 1,
..., n}. Assim, se X é infinito, podemos construir uma família de
injeções de {0, 1, ..., n} em X que não são sobrejetivas. Chame-as de
f_n. Construa a cópia de N dentro de X por indução: comece com 0 -
f_0(0). Daí, 1 -  f_1(1) ou f_1(0), pelo menos um deles é diferente de
f_0(0) porque são ambos diferentes entre si. Em seguida, suponha que
você já definiu até n-1 -  alguma coisa em X, e quer definir a de n.
Podemos supor que f_n(n) é diferente de todas as imagens anteriores
(porque a imagem de {0, 1, ..., n} por f_n tem cardinal n+1, que é
maior que n) e assim n -  f_n(n). Se você preferir, diga que n -
algum elemento de f_n({0, 1, , n}) \ {elementos já utilizados},
que é de cardinal= (n+1) - n = 1. Isso prova que todo conjunto
infinito contém uma cópia de N. Para os puristas, isso usa o axioma da
escolha... para quem gosta da wikipédia, a definição de infinito
como possui uma função injetiva e não bijetiva foi dada pelo
Dedekind, e é interessante em si porque é independente de N ser o
menor conjunto infinito que existe.

Abraços,




Re: [obm-l] ajuda em geometria

2011-12-12 Por tôpico Carlos Nehab

Oi, Marcone,

Os triângulos ABE e ADC são iguais [dois lados iguais (aos lados b=AC e 
c=AB), e ângulo entre tais lados também iguais ... (a A + 60)].

Logo os terceiros lados, BE e DC são também iguais.

Abraços,
Nehab (saudoso da lista e infelizmente sem muito tempo... mas agora vai 
mudar, espero!)


On 12/12/2011 19:11, marcone augusto araújo borges wrote:
   Sobre os lados  AB e AC de um triangulo escaleno ABC são 
construidos dois triangulos equilateros exteriores com vertices em D e 
E,respectivamente.Mostre q  o comprimento de DC é igual ao comprimento 
de BE.







Re: [obm-l] outra questao!!! geometria!

2011-10-30 Por tôpico Carlos Nehab

Bela solução!

Nehab


On 30/10/2011 01:40, Márcio Pinheiro wrote:

Olá,
Não sei exatamente o que você quer dizer com solução plana (seria 
*solução sintética*?), mas fizemos uma solução para o item b que, 
apesar de utilizar alguma Geometria Analítica, tal seria perfeitamente 
dispensável.

O endereço é
http://www.grupoideal.com.br/idealmilitar/pdf/gab_ime_mat_2012_9.pdf
Espero ter ajudado.
Márcio Pinheiro.


Date: Fri, 28 Oct 2011 14:08:48 -0200
From: douglas.olive...@grupoolimpo.com.br
To: obm-l@mat.puc-rio.br
Subject: [obm-l] outra questao!!! geometria!

Olá eu estava tentando achar uma solucao plana para a questao numero 9 
letra b, do ime deste ano 2011, de geometria. a questao é a seguinte:
 9) Considere uma reta r que passa pelo ponto P(2,3). A reta r 
intercepta a curva

*xˆ*2 – 2*xy * – *yˆ*2 = 0 nos pontos A e B. Determine:
a) o lugar geométrico definido pela curva;
b) a(s) possível(is) equação(ões) da reta r, sabendo que (PA)(PB)=17
Na letra (a) o lugar geométrico dará duas retas perpendiculares que 
serão y=(raiz(2)-1)x   (r)  e y=(-raiz(2)-1)x (s)
Na resposta da b são 4 retas fazendo por analitica y=3,x=2 , y=-x+5 e 
y=x+1 então percebi que as duas primeiras são perpendiculares e as 
duas últimas tambem, entao logo vi que se as retas são perpendiculares 
a única solução será essas 4 retas!!
entao estou tentando mostrar que essas retas fazem um angulo reto 
entre elas, mas esta complicado!!!

qualquer ajudinha é bem vinda!! obrigado
Douglas Oliveira.




Re: [obm-l] Curvas e Equações

2011-09-17 Por tôpico Carlos Nehab

Ué, Felipe!

Nenhuma restrição de grau? De nada?
Acho que não entendi sua pergunta, pois há uma infinidade de curvas que 
passam por quaisquer n pontos dados.
Por exemplo a curva (x-x1)(x-x2)...(x-xn) = (y-y1)(y-y2)...(y-yn) 
passa pelos pontos (x1, y1), (x2, y2),...(xn, yn).
Melhor ainda, passa por todos os pontos com x = qq uma das n abscissas e 
y = qq uma das n ordenadas dos pontos dados.

Viajei?

Abraços,
Nehab

Em 16/9/2011 11:40, luiz silva escreveu:

Prezados,
Alguém sabe se exsite algum teorema que defina as condições para que, 
dado um conjunto de n pontos (no R2, por exemplo), exsita uma equação 
para a curva Cx,y que passe por estes pontos.

Abs
Felipe





Re: [obm-l] RE: [obm-l] Limite difícil

2011-09-11 Por tôpico Carlos Nehab

Perfeito, João,

E como o Eduardo também já pontuou nem precisou do senx/x...

Abraços
Nehab

Em 10/9/2011 14:13, João Maldonado escreveu:


v²+c²  = c²/cosk

c(  (v² + c²)^(1/2) - c)/v²  =   ( c²(1-cos)/cos)   / (c²sen²/cos²) = 
 (1-cos).cos/ sen² = (1-cos).cos/(1-cos²)  =  cos/(1+cos)



Como  k- 0,  cosk -  1,  cos/(1+cos) = 1/2

Está certo?
[]'s

João


Date: Sat, 10 Sep 2011 08:31:40 -0300
From: ne...@infolink.com.br
To: obm-l@mat.puc-rio.br
Subject: Re: [obm-l] Limite difícil

Oi, João.

Seu limite tem forte apelo geométrico, pois extrair a raiz quadrada 
de  soma de quadradaos remete para triângulos retângulos...(catetos c 
e v).
Assim, uma simples troca de variável resolve o problema sem 
necessidsde de recursos adicionais além do limite clássico senx/x 
tende a 1 qdo x tende a zero...

Faça v = c.tg(teta) e seu limite se tornará trivial, na variável teta.

Nehab

Em 7/9/2011 20:22, João Maldonado escreveu:

Como posso provar que o limite:



c(   ( v^2 + c^2) ^(1/2) - c)/v^2  = 1/2,  quando v- 0?


[]s
João






Re: [obm-l] Limite difícil

2011-09-10 Por tôpico Carlos Nehab

Oi, João.

Seu limite tem forte apelo geométrico, pois extrair a raiz quadrada 
de  soma de quadradaos remete para triângulos retângulos...(catetos c e v).
Assim, uma simples troca de variável resolve o problema sem necessidsde 
de recursos adicionais além do limite clássico senx/x tende a 1 qdo x 
tende a zero...

Faça v = c.tg(teta) e seu limite se tornará trivial, na variável teta.

Nehab

Em 7/9/2011 20:22, João Maldonado escreveu:

Como posso provar que o limite:



c(   ( v^2 + c^2) ^(1/2) - c)/v^2  = 1/2,  quando v- 0?


[]s
João




[obm-l] Re: [obm-l] Re: [obm-l] RE: [obm-l] Re: [obm-l] Fórmula alternativa(equação do segundo grau)

2011-08-08 Por tôpico Carlos Nehab

Oi, Ralph,

Adorei principalmente o não é Báskara em lugar nenhum do mundo.
O Vitor (que tb está aqui na lista) fica furioso com esta associação 
idiota de vários livros no Brasil...
Alguém ouviu o galo cantar (errado) e saiu repetindo esta bobagem há 
anos por ai.


Abração
Nehab

Em 7/8/2011 20:33, Ralph Teixeira escreveu:
Eu pensei numa relacao bacana, usando a formula quadratica usual, nao 
sei se voce vai gostar.
Vamos resolver ax^2+bx+c=0. Vamos supor que x=0 nao eh uma raiz (ou 
seja, suponha c0); entao, dividindo por x^2, vem

a+b/x+c/x^2=0
Isto quer dizer que 1/x eh raiz da quadratica P(z)=cz^2+bz+a. Se voce 
aplicar a formula quadratica (que nao eh Baskara em lugar nenhum do 
mundo), vem z=[-b+-sqrt(b^2-4ac)]/2c. Como x=1/z... acabou.
(No fundo no fundo, foi isso que o Joao fez, mas ele desenvolveu as 
contas mais completamente)
Diga-se de passagem, a formula alternativa NAO funciona bem quando c=0 
-- a formula vira uma coisa do tipo 0/0, e a gente fica sem achar as 
raizes.

Abraco,
   Ralph

2011/8/7 marcone augusto araújo borges marconeborge...@hotmail.com 
mailto:marconeborge...@hotmail.com


Certo,Bruno.Mas eu queria ver uma maneira de construir a
expressão desse´´x´´.Obrigado.


From: bfr...@gmail.com mailto:bfr...@gmail.com
Date: Sun, 7 Aug 2011 01:45:00 -0300
Subject: [obm-l] Re: [obm-l] Fórmula alternativa(equação do
segundo grau)
To: obm-l@mat.puc-rio.br mailto:obm-l@mat.puc-rio.br

Basta você substituir esse x na equação original e verificar que
vc chega numa expressão válida.

--
Bruno FRANÇA DOS REIS

msn: brunoreis...@hotmail.com mailto:brunoreis...@hotmail.com
skype: brunoreis666
tel: +55 11 9961-7732 tel:%2B55%2011%209961-7732

http://brunoreis.com http://brunoreis.com/
http://brunoreis.com/tech (en)
http://brunoreis.com/blog (pt)

GPG Key: http://brunoreis.com/bruno-public.key

e^(pi*i)+1=0


2011/8/7 marcone augusto araújo borges
marconeborge...@hotmail.com mailto:marconeborge...@hotmail.com

Eu vi em um site, sugerido aqui nessa lista há um bom tempo, a
fórmula

x = 2c/(-b + - raiz(b^2 - 4ac)),para achar as raízes de uma
equação do segundo grau
A demonstração dessa fórmula pode ser feita usando a fórmula
mais conhecida,racionalizando o seu numerador
Desculpem a simplicidade da questão,mas eu gostaria de saber
se há outra maneira de demonstrar essa fórmula alternativa
Agradeço desde já
Abraços,Marcone.







Re: [obm-l] Re: [obm-l] Números Primos

2011-08-06 Por tôpico Carlos Nehab

Bolas,

Esqueci de dizer que M é o N descartado seu último algarismo...
Desculpem-me.

Nehab


Em 5/8/2011 23:02, Carlos Nehab escreveu:

Oi, Regis,

Não lembro do referido email, mas a propriedade a seguir (cuja 
demonstração será um bom exercício para satisfazer sua curiosidade) o 
ajude, no caso de divisibilidade por primos maiores que 5. Embora haja 
critérios outros de divisibilidade (por exemplo por 7 ou 11) acho que 
você vai gostar...


Abraços e bom proveito,
Nehab


Notação: a | b indica a divide b.

Se p é primo, determine inicialmente q, o menor múltiplo positivo de p 
terminado em 1 ou 9 (se p = 17, por exemplo, q = 51).
Naturalmente sempre existirá tal q (um primo impar tem que terminar em 
1, 3, 7 ou 9).


Caso 1.
Se o último dígito de q é 1, então,
p | N  sss p |  (M -  a.r) , onde a é o número que sobra de q quando 
tiramos o 1 (no caso de 17, o 5);


Caso 2.
Se o último dígito de q é 9, então,
p | N  sss p |  [M +  (a+1).r] , onde a é o número que sobra de q 
quando tiramos o 9;


Usando recorrentemente esta propriedade para ir diminuindo o 
dividendo...voce tem ai um procedimento interessante e facilmente 
programável,


Tabelinha
Indicamos nesta ordem, o primo p, o valor de q,  o valor de a e a 
pro­priedade...


pq  a(p | N) sss p divide...
---
7211M - 2r
11  111M - r
13  393M + (3+1)r   = M + 4r
17  515M - 5r
23  696M + (6+1)r  = M + 7r
29  292M + (2+1)r = M + 3r
31  313M - 3r
37  111  11  M - 11r
41  414M - 4r
43  129  12  M + 13r
47  141  14  M  - 14r
etc
===

Em 3/8/2011 15:12, regis barros escreveu:


Boa Tarde Pessoal

Gostaria algum material sobre criterio de divisibilidade que nesta 
lista mandou algum tempo atrás sobre o assunto e do qual não estou 
encontrando o email com o link sobre o assunto.


Regis Godoy Barros

Graduado em Licenciatura em Fisica - IFSP

Graduando em Licenciatura em Matemática - UNICAMP







Re: [obm-l] Re: [obm-l] Números Primos

2011-08-05 Por tôpico Carlos Nehab

Oi, Regis,

Não lembro do referido email, mas a propriedade a seguir (cuja 
demonstração será um bom exercício para satisfazer sua curiosidade) o 
ajude, no caso de divisibilidade por primos maiores que 5. Embora haja 
critérios outros de divisibilidade (por exemplo por 7 ou 11) acho que 
você vai gostar...


Abraços e bom proveito,
Nehab


Notação: a | b indica a divide b.

Se p é primo, determine inicialmente q, o menor múltiplo positivo de p 
terminado em 1 ou 9 (se p = 17, por exemplo, q = 51).
Naturalmente sempre existirá tal q (um primo impar tem que terminar em 
1, 3, 7 ou 9).


Caso 1.
Se o último dígito de q é 1, então,
p | N  sss p |  (M -  a.r) , onde a é o número que sobra de q quando 
tiramos o 1 (no caso de 17, o 5);


Caso 2.
Se o último dígito de q é 9, então,
p | N  sss p |  [M +  (a+1).r] , onde a é o número que sobra de q quando 
tiramos o 9;


Usando recorrentemente esta propriedade para ir diminuindo o 
dividendo...voce tem ai um procedimento interessante e facilmente 
programável,


Tabelinha
Indicamos nesta ordem, o primo p, o valor de q,  o valor de a e a 
pro­priedade...


pq  a(p | N) sss p divide...
---
7211M - 2r
11  111M - r
13  393M + (3+1)r   = M + 4r
17  515M - 5r
23  696M + (6+1)r  = M + 7r
29  292M + (2+1)r = M + 3r
31  313M - 3r
37  111  11  M - 11r
41  414M - 4r
43  129  12  M + 13r
47  141  14  M  - 14r
etc
===

Em 3/8/2011 15:12, regis barros escreveu:


Boa Tarde Pessoal

Gostaria algum material sobre criterio de divisibilidade que nesta 
lista mandou algum tempo atrás sobre o assunto e do qual não estou 
encontrando o email com o link sobre o assunto.


Regis Godoy Barros

Graduado em Licenciatura em Fisica - IFSP

Graduando em Licenciatura em Matemática - UNICAMP





Re: [obm-l] A procura de um livro! (off-topic)

2011-07-20 Por tôpico Carlos Nehab

Tem na Amazon, João,

Abraços,
Nehab

Em 20/7/2011 08:13, Pedro Júnior escreveu:

Alguém poderia me indicar algum site que tenha o livro:
L. E. Dickson, Algebras and their Arithmetics, University of Chicago 
Press, 1923
p.s.: poderia ser para download, pois pela data acho que não tem mais 
para vender!

--

Pedro Jerônimo S. de O. Júnior

Professor de Matemática

Geo João Pessoa – PB






[obm-l] teste OFF Topic

2011-06-29 Por tôpico Carlos Nehab

Teste

Carlos Nehab
=
Instruções para entrar na lista, sair da lista e usar a lista em
http://www.mat.puc-rio.br/~obmlistas/obm-l.html
=


Re: [obm-l] Desigualdade (Como provar?)

2011-06-21 Por tôpico Carlos Nehab

Oi, Paulo.

É simples e clássico.
Basta usar média aritmética = média geométrica em S e S'.

Abraços
Nehab

Em 21/6/2011 08:34, Paulo Argolo escreveu:

Caros Colegas,

Não consegui ainda uma demonstração. Seria possível fazê-la por indução finita?

Abraços do Paulo.
-








Date: Mon, 13 Jun 2011 22:49:41 +0200
Subject: Re: [obm-l] Desigualdade (Como provar?)
From: bernardo...@gmail.com
To: obm-l@mat.puc-rio.br

2011/6/13 Paulo Argoloargolopa...@hotmail.com:

Caros Colegas,
Como podemos provar que, dados n numeros reais positivos (n1), nem todos 
iguais, vale a desigualdade abaixo?

S . S'  n^2 (S é a soma dos n números, S' é a soma dos inversos desses n 
números.)


Tente mostrar isso para n = 2, n = 3, expandindo tudo. Dá poucos
termos, e daí acho que você vai ver como prova para n qualquer.

Abraços,
--
Bernardo Freitas Paulo da Costa

=
Instruções para entrar na lista, sair da lista e usar a lista em
http://www.mat.puc-rio.br/~obmlistas/obm-l.html
=   


=
Instruções para entrar na lista, sair da lista e usar a lista em
http://www.mat.puc-rio.br/~obmlistas/obm-l.html
=



=
Instruções para entrar na lista, sair da lista e usar a lista em
http://www.mat.puc-rio.br/~obmlistas/obm-l.html
=


Re: [obm-l] Blog interessante OFF TOPIC

2011-05-24 Por tôpico Carlos Nehab

Que que é isto, Artur,

Tem neguinho MUITO mais antigo que você.  Fica frio. Você ainda está 
pelo menos na categoria de jovem Braquiossauro (final da era mesozóica).
Mas que temos pelo menos dois colegas da época do Big Bang, a, isso 
temos!
Como não tenho autorização de ninguém para declinar a idade dos 
dinossauros que já mapeei, registro apenas a minha idade: 65,5 anos 
muito bem vividos, e uma boa parte disso com a maravilhosa companhia de 
muitos de vocês, já tendo tido o prazer de ter sido professor de vários 
de vocês e aluno de alguns...


Agora, para quem gosta de jogo da forca, apenas alguns desafios (dos 
quase 75 que poderia propor)


R _ _ _ H  T _ _ _ _ _ _ A (seu admirador há quase um século)
R _ G _ _ _ _  P_ _ _ E (muito, muito amigo)
A _ _ _ _ _ B _ U _ _ _ _ _  (o admiro e adoramos brigar)
P _ _ _ _ S _ _ T _ R _ _ _  (não adianta ficar zangado comigo)
N _ _ _ _ _U  S _ _ _ _ _ _ _ (um pioneiro - inesquecível)
E _ _ _ _ _ _ W _ _ _ _ R (anda ausente, o geômetra, infelizmente)

Abraços a todos,
Nehab

Em 24/5/2011 10:10, Artur Steiner escreveu:

Eu então sou da época do Big Bang..,

Artur

Enviado de meu telefone Nokia



-Original Message-
From: Carlos Nehab
Sent: 5/24/2011 12:13:45 AM
To: obm-l@mat.puc-rio.br
Subject: Re: [obm-l] Blog interessante
Hahaha,

Se o jovem Carlos Victor já foi promovido a dinossauro e catapultado
para as eras Jurássica/Cretássica, fico até feliz, pois então ainda não
inventaram categoria para minha espécie.
Sou anterior, possivelmente,... hm, deixa eu fazer uma regra de
três, huuum perái  h !
Caraca!  deu período Devoniano (400.000.000 AC) ...
E o bichinho mais simpático daquela época, em meu entendimento, era o
Goniattes, que pode até propiciar alguns probleminhas de matemágica
interessantes.
E como o dito cujo foi achado em Marrocos, terra de um lado de meus
ancestrais, até acho que acertei na regra de três.
Vejam  em
http://pt.wikipedia.org/wiki/Ficheiro:Goniatites_sp.3_-_Devonico.JPG
Não sou uma linda espiral?
Portanto, desafio vocês a postar problemas legais envolvendo espirais
interessantes no seu Site, de nivel médio e superior...
Os dinossauros não os ajudarão, mas se sentirão confortados e honrados.

Abraços e Sucesso!
Nehab

PS:
Caso vocês não saibam, pois são ainda jovens, há aproximadamente 48
dinossauros na lista, já mapeados por mim e uns 26 pré-Jurássicos também...
Ou vocês pensaram que eu e o Carlos Victor estávamos sozinhos?


Em 22/5/2011 23:03, DadosDeDeus Blog escreveu:

Caros companheiros,

pedimos desculpas pelo incômodo da mensagem (não é spam! rs), mas
gostaríamos de convidá-los a acessar o blog *Dados de Deus *(
http://dadosdedeus.blogspot.com).

Desenvolvido por estudantes das melhores universidades do país, o Dados
consiste em uma página *sem fins lucrativos* que visa à divulgação da
matemática com qualidade em diversos níveis, com postagens desde assuntos
específicos para vestibulares (*IME/ITA*) e *olimpíadas *a tópicos
pertinentes ao *ensino superior*.

Em nosso último post publicamos duas formas interessantes de determinar
sin18°, uma utilizando a geometria do pentágono regular e outra algébrica.
Agradecimentos aliás aos professores Nehab e Victor (dinossauros da lista,
rs) que nos mostraram em alguma aula esses belos raciocínios.

Estamos abertos a críticas/sugestões e desde já agradecemos pela atenção.


=
Instruções para entrar na lista, sair da lista e usar a lista em
http://www.mat.puc-rio.br/~obmlistas/obm-l.html
=



=
Instruções para entrar na lista, sair da lista e usar a lista em
http://www.mat.puc-rio.br/~obmlistas/obm-l.html
=


Re: [obm-l] Blog interessante

2011-05-23 Por tôpico Carlos Nehab

Hahaha,

Se o jovem Carlos Victor já foi promovido a dinossauro e catapultado 
para as eras Jurássica/Cretássica, fico até feliz, pois então ainda não 
inventaram categoria para minha espécie.
Sou anterior, possivelmente,... hm, deixa eu fazer uma regra de 
três, huuum perái  h !

Caraca!  deu período Devoniano (400.000.000 AC) ...
E o bichinho mais simpático daquela época, em meu entendimento, era o 
Goniattes, que pode até propiciar alguns probleminhas de matemágica 
interessantes.
E como o dito cujo foi achado em Marrocos, terra de um lado de meus 
ancestrais, até acho que acertei na regra de três.
Vejam  em 
http://pt.wikipedia.org/wiki/Ficheiro:Goniatites_sp.3_-_Devonico.JPG

Não sou uma linda espiral?
Portanto, desafio vocês a postar problemas legais envolvendo espirais 
interessantes no seu Site, de nivel médio e superior...

Os dinossauros não os ajudarão, mas se sentirão confortados e honrados.

Abraços e Sucesso!
Nehab

PS:
Caso vocês não saibam, pois são ainda jovens, há aproximadamente 48 
dinossauros na lista, já mapeados por mim e uns 26 pré-Jurássicos também...

Ou vocês pensaram que eu e o Carlos Victor estávamos sozinhos?


Em 22/5/2011 23:03, DadosDeDeus Blog escreveu:

Caros companheiros,

pedimos desculpas pelo incômodo da mensagem (não é spam! rs), mas
gostaríamos de convidá-los a acessar o blog *Dados de Deus *(
http://dadosdedeus.blogspot.com).

Desenvolvido por estudantes das melhores universidades do país, o Dados
consiste em uma página *sem fins lucrativos* que visa à divulgação da
matemática com qualidade em diversos níveis, com postagens desde assuntos
específicos para vestibulares (*IME/ITA*) e *olimpíadas *a tópicos
pertinentes ao *ensino superior*.

Em nosso último post publicamos duas formas interessantes de determinar
sin18°, uma utilizando a geometria do pentágono regular e outra algébrica.
Agradecimentos aliás aos professores Nehab e Victor (dinossauros da lista,
rs) que nos mostraram em alguma aula esses belos raciocínios.

Estamos abertos a críticas/sugestões e desde já agradecemos pela atenção.



=
Instruções para entrar na lista, sair da lista e usar a lista em
http://www.mat.puc-rio.br/~obmlistas/obm-l.html
=


[obm-l] Re: [obm-l] Re: [obm-l] Um problema curioso e... insolúvel OFF TOPIC

2011-05-19 Por tôpico Carlos Nehab

Hahaha,

Adorei Bruno!
Este negócio de andar (nadar) prá frente, para trás, girar, etc, etc, me 
fez fazer uma viagem no tempo, pois me lembrei do velho LOGO ainda em DOS!
Como não sei sua idade, posso estar falando japonês, mas há 1 
anos atrás (como diria o Raul Seixas), quando a IBM encampou um 
interessante projeto de Logo nas escolas, minha empresa (na época) era 
chancelada para apresentar treinamentos desta (boa) geringonça aos 
professores.  O velho e eficaz construtivismo ainda pouco usado nas 
escolas, mesmo hoje (neguinho ainda anda muito conteudista pro meu gosto).


Se não estou delirando, acho que na época ainda havia muito Windows 
3.11... na praça (mas certamente eu já era viciado no malditoTetris 
usual e em uma versão tridimensional ótima).


Caraca!  Que viagem!

Afetuoso abraço,
Nehab

Em 19/5/2011 17:23, Bruno França dos Reis escreveu:

Em aberto?

Se o nadador estivesse nadando paralelo ao rio, é só ele fazer uma curva
mínima, e continuar até chegar às margens.

Caso o nadador não saiba a direção em que estava nadando (suponhamos uma
briga com os peixes, que o deixou desorientado, antes de ter seus olhos
devorados), ele poderia nadar seguindo uma espiral, aí certamente
encontrará a margem, não? O algoritmo seria:

n- 1
Enquanto não achar a margem, repita:
  - dar n braçadas para frente
  - virar 90 graus para a esquerda
  - dar n braçacas para frente
  - virar 90 graus para a esquerda
  - n- n + 1

Como a largura é finita, e a espiral cresce de tamanho em todas as direções,
esse algoritmo certamente termina em um tempo finito!

Tem alguma falha que eu não vi nesse processo?

Abraço!
Bruno


--
Bruno FRANÇA DOS REIS

msn: brunoreis...@hotmail.com
skype: brunoreis666
tel: +55 11 9961-7732

http://brunoreis.com
http://brunoreis.com/tech (en)
http://brunoreis.com/blog (pt)

GPG Key: http://brunoreis.com/bruno-public.key

e^(pi*i)+1=0


2011/5/19 Albert Bouskelabousk...@msn.com


Olá a todos,



Uma curiosidade: – Parece-me que o problema abaixo (tão simples!) permanece
em aberto.



Um nadador está nadando (o que mais pode fazer um nadador?) em um ponto
qualquer de um rio horizontal, retilíneo, com correnteza desprezível,
comprimento infinito e largura finita.



Subitamente, peixes extremamente vorazes devoram os olhos do malfadado
nadador, ou, com menos drama, cai a noite absolutamente escura.



Qual é a trajetória que o nadador deve trilhar, i.e., nadar, para atingir –
seguramente – uma das margens, nadando a menor distância possível?



Obs.: – O malfadado nadador tem, implantado em sua cabeça, um sistema de
navegação que lhe informa, continuamente, a sua posição em relação ao ponto
inicial (o ponto no qual os peixes devoraram os seus olhos).



Saudações,

Albert Bouskela

bousk...@msn.com





=
Instru��es para entrar na lista, sair da lista e usar a lista em
http://www.mat.puc-rio.br/~obmlistas/obm-l.html
=


Re: [obm-l] Fwd: Identidade de Euler OFF TOPIC

2011-05-05 Por tôpico Carlos Nehab

Oi, João,

Infelizmente ando trabalhando muito. Mas é apenas uma fase.
Já já volto a ser pais participante!

Um forte abraço
Nehab

Em 28/4/2011 18:01, João Luís Guimarães escreveu:

Por onde você anda, Nehab? Tá muito sumido aqui da lista, você e suas
interessantes intervenções!

João Luís

Em 28 de abril de 2011 17:21, Carlos Nehabne...@infolink.com.br  escreveu:


Oi, Fábio,

Não resisti:

Resolva os seguinte problema de duas maneiras (uma técnica básica e útil
para resolver identidades deste tipo).
De quantas maneira posso formar comissões de p pessoas, a partir de um
total de m + n pessoas, sendo m o total de pessoas que moram no Maracanã e n
as pessoas que moram em Nilópolis?

Abraços,
Nehab

Em 28/4/2011 13:24, fabio henrique teixeira de souza escreveu:

  -- Mensagem encaminhada --

De: fabio henrique teixeira de souzafabiodja...@ig.com.br
Data: 28 de abril de 2011 08:52
Assunto: Identidade de Euler
Para: obm-l@mat.puc-rio.br


Pessoal, estou batendo cabeça e não consigo demonstrar que
C(m,0).C(n,p) + C(m,1).C(n,p-1) + C(m,2).C(n,p-2) + ... + C(m,p).C(n,0) =
C(m+n,p)

Alguém pode me dar uma dica?



=
Instruções para entrar na lista, sair da lista e usar a lista em
http://www.mat.puc-rio.br/~obmlistas/obm-l.html
=



=
Instruções para entrar na lista, sair da lista e usar a lista em
http://www.mat.puc-rio.br/~obmlistas/obm-l.html
=


Re: [obm-l] Fwd: Identidade de Euler (OFFTOPIC)

2011-05-01 Por tôpico Carlos Nehab

Oi, querido amigo.
Grande abraço
Nehab

Em 28/4/2011 17:40, Carlos Victor escreveu:

Oi  Mestre  Nehab ,
Gostei da sugestão e mais ainda  das n pessoas que moram em Nilópolis (
minha  terrinha).

Abraços

Carlos  Victor

Em 28 de abril de 2011 17:21, Carlos Nehabne...@infolink.com.br  escreveu:


Oi, Fábio,

Não resisti:

Resolva os seguinte problema de duas maneiras (uma técnica básica e útil
para resolver identidades deste tipo).
De quantas maneira posso formar comissões de p pessoas, a partir de um
total de m + n pessoas, sendo m o total de pessoas que moram no Maracanã e n
as pessoas que moram em Nilópolis?

Abraços,
Nehab

Em 28/4/2011 13:24, fabio henrique teixeira de souza escreveu:

-- Mensagem encaminhada --

De: fabio henrique teixeira de souzafabiodja...@ig.com.br
Data: 28 de abril de 2011 08:52
Assunto: Identidade de Euler
Para: obm-l@mat.puc-rio.br


Pessoal, estou batendo cabeça e não consigo demonstrar que
C(m,0).C(n,p) + C(m,1).C(n,p-1) + C(m,2).C(n,p-2) + ... + C(m,p).C(n,0) =
C(m+n,p)

Alguém pode me dar uma dica?



=
Instruções para entrar na lista, sair da lista e usar a lista em
http://www.mat.puc-rio.br/~obmlistas/obm-l.html
=



=
Instruções para entrar na lista, sair da lista e usar a lista em
http://www.mat.puc-rio.br/~obmlistas/obm-l.html
=


Re: [obm-l] Fwd: Identidade de Euler

2011-04-28 Por tôpico Carlos Nehab

Oi, Fábio,

Não resisti:

Resolva os seguinte problema de duas maneiras (uma técnica básica e útil 
para resolver identidades deste tipo).
De quantas maneira posso formar comissões de p pessoas, a partir de um 
total de m + n pessoas, sendo m o total de pessoas que moram no Maracanã 
e n as pessoas que moram em Nilópolis?


Abraços,
Nehab

Em 28/4/2011 13:24, fabio henrique teixeira de souza escreveu:

-- Mensagem encaminhada --
De: fabio henrique teixeira de souzafabiodja...@ig.com.br
Data: 28 de abril de 2011 08:52
Assunto: Identidade de Euler
Para: obm-l@mat.puc-rio.br


Pessoal, estou batendo cabeça e não consigo demonstrar que
C(m,0).C(n,p) + C(m,1).C(n,p-1) + C(m,2).C(n,p-2) + ... + C(m,p).C(n,0) =
C(m+n,p)

Alguém pode me dar uma dica?



=
Instruções para entrar na lista, sair da lista e usar a lista em
http://www.mat.puc-rio.br/~obmlistas/obm-l.html
=


Re: [obm-l] Geometria

2011-04-28 Por tôpico Carlos Nehab

Oi, João,

O seu exercício é um clássico.
Ai vai a dica. Um trabalho legal da Silvana: você vai gostar.

http://www.mat.puc-rio.br/~hjbortol/complexidade/complexidade-em-geometria.pdf

Capítulo 2 a partir da página 28
Olhe também a página 36.

Abraços,
Nehab

Em 26/4/2011 20:22, João Maldonado escreveu:

O seguinte problema está no livro  Geometria I de Morgado, e não sei porque  
não estou conseguindo resolvê-lo. Sei que a resposta é 30º, se alguém  puder 
ajudar fico grato.

Em um triângulo isósceles ABC, se base BC, o ângulo  vale 20º. P é um ponto 
sobre AB tal que o ângulo PCB = 60º. Q é um ponto em AC tal que QBC = 50º. Qual 
a medida do ângulo CPQ?
[]'sJoão


=
Instruções para entrar na lista, sair da lista e usar a lista em
http://www.mat.puc-rio.br/~obmlistas/obm-l.html
=


Re: [obm-l] Divisibilidade por 13

2010-12-29 Por tôpico Carlos Nehab

Oi, Olavo e Felipe,

Segue um resumo adaptado de http://www.egge.net/~savory/maths1.htm, 
escrito há muito tempo por mim e baseado nessa referência, que eu sugeri 
em e-mail anterior.


Seja N um inteiro, r seu último dígito e M o número formado pelos 
algarismos anteriores (por exemplo, se N = 3249, então r = 9 e M = 324).


a) Exemplo preliminar (divisibilidade 17)

Propriedade
17 | N se e somente se  17 | M - 5r

Exemplos

N = 2.343
17 | 2343   sss  17 | ( 234  - 5.3)   sss  17 | 219  sss
17 | 21 - 5x9   sss  17 |  -24;
logo, 2343 não é divisível por 17.

N = 15.912
17 | 15912   sss  17 | (1591 - 5.2)   sss   17 | 1581   sss
17 | (158 - 5.1)  sss 17 |  153  sss 17 | (15 - 5.3)  sss
17 | 0; logo, 17 | 15912.

b) Caso geral
Se p é primo, determine q, o menor múltiplo positivo de p terminado em 1 
ou 9 (se p = 17  então q = 51).


i) Se o último dígito de q = 1:

p | N  sss p |  M -  ar , onde a é o número que sobra de q quando 
tiramos o 1 (no caso de 17, o 5);


ii) Se o último dígito de q = 9:
p | N  sss p |  M +  (a+1) r , onde a é o número que sobra de q quando 
tiramos o 9;


c) Tabelinha
Veja a tabela abaixo, onde indicamos nesta ordem, o primo p, o valor de 
q,  o valor de a e a pro¬priedade...


pq  a   (p | N) sss p divide...
 7   21 1M - 2r
11   11 1M - r
13   39 3M + (3+1)r   = M + 4r
17   51 5M - 5r
23   69 6M + (6+1)r  = M + 7r
29   29 2M + (2+1)r = M + 3r
31   31 3M - 3r
37  11111M - 11r
41   41 4M - 4r
43  12912M + 13r
47  14114M - 14r
...

As demostrações são simples, mas qualquer dúvida escreva.

Abraços,
Nehab


Em 20/12/2010 09:35, Antonio Neto escreveu:
   Senhores, permitam meter a colher torta. Com a mesma notação do 
texto, um outro possível critério é: n = 10x + a é divisível por 13 
se, e somente se, x + 4a o for. Note que vc multiplica o algarismo 
final por -9, e eu por 4. Ahá!!! 4-(-9) = 13. Experimente também x + 
17a, etc... Há um livrinho russo, da Editora Mir, o exemplar que tenho 
está em espanhol, chamado Criterios de divisibilidad, acho que é do 
Vorobiov, mas não estou em casa agora. Divirta-se, abraços, olavo.


Antonio *Olavo* da Silva Neto





Date: Fri, 17 Dec 2010 11:54:57 -0200
From: ne...@infolink.com.br
To: obm-l@mat.puc-rio.br
Subject: Re: [obm-l] Divisibilidade por 13

Oi, Felipe,

Você vai gostar de
http://www.egge.net/~savory/maths1.htm 
http://www.egge.net/%7Esavory/maths1.htm


Seu caso é equivalente ao que o texto menciona. Procure perceber isto.

Abraços,
Nehab


Em 16/12/2010 23:55, Felipe Diniz escreveu:

n = 10x+a, a entre 0 e 9.

x-9a = 0 mod13
entao x=9a mod13

n= 10x+a = 91a = 13*7a = 0  mod 13

2010/12/16 marcone augusto araújo borges
marconeborge...@hotmail.com mailto:marconeborge...@hotmail.com

Dado um número,8281,por exemplo.Fazendo 828 - 9*1=819 e
repetindo o procedimento:81 - 9*9=0
zero é divisível por 13,logo8281 também é.
Para 867:86 - 9*7=23.
23 não é divisível por 13,logo 867 também não é.
Como provar que a regra é verdadeira?







Re: [obm-l] Divisibilidade por 13

2010-12-17 Por tôpico Carlos Nehab

Oi, Felipe,

Você vai gostar de
http://www.egge.net/~savory/maths1.htm

Seu caso é equivalente ao que o texto menciona. Procure perceber isto.

Abraços,
Nehab


Em 16/12/2010 23:55, Felipe Diniz escreveu:

n = 10x+a, a entre 0 e 9.

x-9a = 0 mod13
entao x=9a mod13

n= 10x+a = 91a = 13*7a = 0  mod 13

2010/12/16 marcone augusto araújo borges marconeborge...@hotmail.com 
mailto:marconeborge...@hotmail.com


Dado um número,8281,por exemplo.Fazendo 828 - 9*1=819 e repetindo
o procedimento:81 - 9*9=0
zero é divisível por 13,logo8281 também é.
Para 867:86 - 9*7=23.
23 não é divisível por 13,logo 867 também não é.
Como provar que a regra é verdadeira?






Re: [obm-l] Teoria dos números

2010-12-17 Por tôpico Carlos Nehab

Marcone,

Este é simples, desde que você saiba justificar o fato de a^n - 1 ser 
divisível por a - 1 (a diferente de 1).

Qual série você cursa atualmente?

Tais primos são chamados de primos de Mersenne e os maiores primos até 
hoje descobertos são desta forma.

Veja em
http://pt.wikipedia.org/wiki/Primo_de_Mersenne
e
http://primes.utm.edu/
que é a página oficial dos maiores primos...



Abraços,
Nehab

Em 16/12/2010 23:36, marcone augusto araújo borges escreveu:
Mostre que se a e n são inteiros positivos,com n = 2 ,tais que a^n - 
1 é primo,então necessariamente a = 2 e n é primo. 




Re: [obm-l] calculo sem calculadora

2010-12-04 Por tôpico Carlos Nehab
Ou, bolas, calculando cos 2x = [1 - 2(senx^)2]/2 pra ver se é um ângulo 
mais bonitinho e...o, vai ser!

Nehab

Em 4/12/2010 02:07, Lucas Colucci escreveu:

Se senx=(sqrt(6)-sqrt(2))/4, cosx=(sqrt(6)+sqrt(2))/4 (supondo 0xpi/2).

Assim, sen2x=2senxcosx=1/2 = 2x=pi/6 ou 2x=5pi/6 = x=pi/12 ou 
x=5pi/12. Como 5pi/12pi/4, tg(5pi/12)1 = sen(5pi/12)cos(5pi/12), 
o que não ocorre para os nossos valores de senx e cosx. Assim, x=pi/12 
é a solução.



Lucas Colucci

Date: Fri, 3 Dec 2010 22:26:23 -0300
Subject: Re: [obm-l] calculo sem calculadora
From: e-...@ig.com.br
To: obm-l@mat.puc-rio.br

Então façamos na ordem inversa:
sen(x) = [R(6) - R(2)]/4 = R(6)/4 - R(2)/4 = [R(2)/2] * [R(3)/2] - 
[R(2)/2] * [1/2] = sen45º * cos30º - sen30º * cos45º = sen(45º - 30º) 
= sen15º.

Logo, x = 15º

Em 3 de dezembro de 2010 13:41, Felipe Diniz edward.elric.br 
http://edward.elric.br@gmail.com http://gmail.com escreveu:


Não, é oq ele queria, voce ja partiu da resposta.

2010/12/3 Vitor Alves vitor__r...@hotmail.com
mailto:vitor__r...@hotmail.com

voce tem que

sen(45-30)=sen45.cos30-sen(30).cos(45)=raiz(2)/2.raiz(3)/2-raiz(2)/2.1/2=[raíz(6)-raíz(2)]/4


Date: Fri, 3 Dec 2010 13:54:29 -0200
Subject: [obm-l] calculo sem calculadora
From: teliog...@gmail.com mailto:teliog...@gmail.com
To: obm-l@mat.puc-rio.br mailto:obm-l@mat.puc-rio.br

Boa tarde professores,

gostaria de saber a forma de determinar o valor de x, sabendo
que sen(x) = [RAIZ(6) - RAIZ(2)] ¸4, semusartabelaoucalculadora.
Bem, eu sei que a resposta é x = 15º, mas como encontrar esse
valor algebricamente?

Agradeço a ajuda.

abraços

Thelio Gama








Re: [obm-l] Trigonometria

2010-11-15 Por tôpico Carlos Nehab

Oi, Pedro,

Infelizmente o enunciado está errado.
Mas para você não ficar triste, tente resolver algo parecido e correto:

2cos 20 - 1/ (2cos 40 -1)  é um inteiro...

Abraços
Carlos Nehab

Dica: este negócio de 20, 40 e 80 graus muitas vezes acabam em samba se 
você usar as expressões de arco triplo, pois linhas trigonométricas 
desse arcos se expressam em termos de raízes de uma equação cúbica..., 
que no fundo é o que as expressões do arco triplo nos mostram...  Se 
você conhecer Cardano, poderá inclusive se divertir (?) explicitando os 
senos e cossenos destes arcos.
Vários problemas interessantes já circularam por aqui com estes 
malditos e instigantes ângulos...  Além disso um estudo do eneágono e 
do octadecágono (18 lados) também será fascinante para quem gosta destes 
angulozinhos decididamente desafiadores.



Em 15/11/2010 00:00, Pedro Júnior escreveu:

Parece simples mais ainda não consegui exergar o caminho.
Usei tansformações, forma exponencial dos complexos, combinei várias 
transformações, etc, só ainda não dei um tratamento geométrico..



Vejam: Mostre que 2cos20º - 1/cos80º é um inteiro.


Abraços.

Pedro Júnior
João Pessoa - PB




Re: [obm-l] 0,99999... = 0

2010-10-15 Por tôpico Carlos Nehab

 Oi Antonio Ricardo,

Não consegui ter acesso a este artigo, mas conheço um muito bom:

http://www.math.umt.edu/TMME/vol7no1/TMME_vol7no1_2010_article1_pp.3_30.pdf

Abraços,
Nehab

Em 15/10/2010 14:00, antonio ricardo escreveu:

olá a todos

vasculhando a internet, encontrei por acaso essa afirmação:
0,999... = 0

gostaria que comentassem.

valeu!

o artigo encontra-se aqui:
http://www.dmat.ufrr.br/~gentil/images/stories/Artigos/palestra.pdf 
http://www.dmat.ufrr.br/%7Egentil/images/stories/Artigos/palestra.pdf






Re: [obm-l] x^y = y^x

2010-10-04 Por tôpico Carlos Nehab

 Bernardo,

Você é muito mau...

Nehab

Em 4/10/2010 12:43, Bernardo Freitas Paulo da Costa escreveu:

Paulo: quanto vale i^2i ?

2010/10/4 Paulo Argoloargolopa...@hotmail.com:

Prezados Colegas,

Proponho a questão abaixo.

QUESTÃO:

Determinar, no universo dos números complexos, o conjunto solução da equação
x^y = y^x, sendo x diferente de y.

Desde já, muito grato.

Paulo Argolo









=
Instruções para entrar na lista, sair da lista e usar a lista em
http://www.mat.puc-rio.br/~obmlistas/obm-l.html
=


Re: [obm-l] Hierarquia das Operações

2010-07-28 Por tôpico Carlos Nehab

Oi, Luiz,

Tô ausente da lista há algum tempo justamente por falta de tempo, mas 
gostaria de participar desta ...


Vários colegas já responderam na linha de é uma convenção, tanto faz 
e eu concordo com os comentários já postados.
Mas talvez a chave da questão no que diz respeito ao ensino das notações 
disponíveis para representar expressões seja perceber que, na verdade, 
não é necessário nenhum símbolo separador para escrevê-las sem ambiguidade.


Como a turma de computação aprende, você tem 3 formas usuais chamadas de 
infixa, pósfixa e préfixa para representar expressões.
Algumas máquinas de calcular aceitam a forma préfixa naturalmente, 
também chamada de notação polonesa.
Procure na Web os verbetes citados se quiser detalhes, mas ai dou apenas 
uma idéia da notação polonesa que, se ensinada às crianças, evitaria 
todas esta discussão.


Por exemplo, para escrever expressões com esta notação escreva, 
recursivamente (repetidamente), o símbolo da operação em primeiro lugar 
e, à direita, as duas parcelas a ela associadas, na ordem em que ocorrem:


Exemplinhos:
a + b   será escrito como +ab
a+ b*c será escrito como  +a*bc
a * (b + c)  será escrito como *a+bc
e assim sucessivamente.

Ou eja a notação é chamada de préfica porque a operação precede as 
parcelas a elas associadas. Percebeu?


Ou seja, o uso de parênteses, colchetes e chaves é apenas para facilitar 
o uso da notação usual que utilizamos que é burra e limitada, pois lemos 
da esquerda para a direita mas não necessariamente as operações que 
desejamos realizar sejam calculadas nesta ordem.   Teoricamente, no 
ocidente, portanto, a notação mais sensata é a préfixa (ou polonesa)...  
Ma se eu fosse japones ou árabe, possivelmente (espero não estar dando 
mancada) preferiria a notação pósfixa... :-)


Abraços
Nehab

PS: Só por curiosidade, no meu tempo, há mais ou menos 60 anos, a 
ordem era parênteses primeiro, depois colchetes e finalmente chaves.  
Que bobagem !



Luiz Rodrigues escreveu:

Olá, pessoal!!!
Tudo bem???
Tenho uma dúvida já faz algum tempo e não consegui esclarecê-la.
É sobre a hierarquia das operações (parênteses, depois colchetes etc).
Alguém sabe como isso surgiu? Será que foi um acordo feito entre 
matemáticos?

Muito obrigado!!!
Um abraço para todos.
Luiz.


=
Instruções para entrar na lista, sair da lista e usar a lista em
http://www.mat.puc-rio.br/~obmlistas/obm-l.html
=


Re: [obm-l] Re: [obm-l] Hierarquia das Operaçõ es OFFTOPIC

2010-07-28 Por tôpico Carlos Nehab




Oi, Bernardo (e Luiz)

Voc tem toda razo, mais achei mais natural para o Luiz ele primeiro
se divertir com a notao prefixada.  mais natural.

Na verdade j programei durante MUITOS anos e fui tambm professor na
rea... 
Mas sem querer ser saudosista (j sendo) lembro aos jovens que meu
primeiro PC tinha 64k de memria e meu HD 10 Mb (isto mesmo)...
Alm disso no IME, onde cursei Engenharia Eltrica (1965/1969) - me
formei h mais de 40 anos... o IBM 1130 tinha 8k de memria e no era
muititarefa... Tempos inacreditveis...

E sem nenhum saudosismo bobo, naquela poca programar era mais do que
uma arte. Era mgico.
E isto em linguagens malucas, como Assembler, Mumps, PL1, Pascal /
Turbo Pascal e... at Cobol... (um saco) e... outras linguagens
"exdruxulas" da poca.
Pouco depois do C e da Orientao a Objeto, acabou meu saco e eu me
decidi pela Matemgica... 

Mas na linha da fronteira entre Informtica e Matemtica, tambm
durante anos fui professor de Tcnicas de Construo de Algoritmos
(inclusive Algoritmos Heursticos), Matemtica Discreta, Linguagens
Formais, Complexidade de Computao, etc, para a galera de Cincia da
Computao e, c pr ns, isto  mais matemtica do que informtica,
n...

Portanto, meu comentrio sobre as notaes in/ps/prefixas vm deste
saudoso tempo (1970 a 2000).

Quanto  sua charada, deixo como exerccio pro Luiz, responsvel por
esta discusso...

Grande abrao,
Nehab

PS: Gostosa inveja de voc ai na Frana... Ralando muito ou apenas
passando frias?


Bernardo Freitas Paulo da Costa escreveu:

  2010/7/28 Carlos Nehab ne...@infolink.com.br:
  
  
Oi, Luiz,

T ausente da lista h algum tempo justamente por falta de tempo, mas
gostaria de participar desta ...

  
  Oi Nehab! Eu aproveitei o almoo hoje para perguntar para alguns
colegas franceses se essa histria de "parnteses, colchetes, chaves"
existia por aqui... Parece que no, enfim, ningum se lembra de ter
visto uma coisa dessas na escola.

Mas eu respondo mesmo porque eu gosto de perturbar, e voc falou da
notao polonesa, eu no posso evitar falar da RPN (reverse polish
notation, ou notao polonesa invertida), e tambm propor um
probleminha.
  
  
Vrios colegas j responderam na linha de " uma conveno", "tanto faz" e
eu concordo com os comentrios j postados.
Mas talvez a chave da questo no que diz respeito ao ensino das notaes
disponveis para representar expresses seja perceber que, na verdade, no 
necessrio nenhum smbolo separador para escrev-las sem ambiguidade.

Como a turma de computao aprende, voc tem 3 formas usuais chamadas de
infixa, psfixa e prfixa para representar expresses.
Algumas mquinas de calcular aceitam a forma prfixa naturalmente, tambm
chamada de notao polonesa.
Procure na Web os verbetes citados se quiser detalhes, mas ai dou apenas uma
idia da "notao polonesa" que, se ensinada s crianas, evitaria todas
esta discusso.

Por exemplo, para escrever expresses com esta notao escreva,
recursivamente (repetidamente), o smbolo da operao em primeiro lugar e, 
direita, as duas parcelas a ela associadas, na ordem em que ocorrem:

Exemplinhos:
a + b  ser escrito como +ab
a+ b*c ser escrito como +a*bc
a * (b + c) ser escrito como *a+bc
e assim sucessivamente.

Ou eja a notao  chamada de prfica porque a operao precede as parcelas
a elas associadas. Percebeu?

Ou seja, o uso de parnteses, colchetes e chaves  apenas para facilitar o
uso da notao usual que utilizamos que  burra e limitada, pois lemos da
esquerda para a direita mas no necessariamente as operaes que desejamos
realizar sejam calculadas nesta ordem.  Teoricamente, no ocidente,
portanto, a notao mais sensata  a prfixa (ou polonesa)... Ma se eu
fosse japones ou rabe, possivelmente (espero no estar dando mancada)
preferiria a notao psfixa... :-)

  
  
E se voc fosse programador, talvez preferisse que seus utilizadores
usassem a notao posfixa, porque usa menos memria e as contas so
feitas diretamente na pilha, o que simplifica bastante o procedimento,
e deve acelerar um pouquinho as coisas ! De um ponto de vista
funcional (de funo, no que seja mais simples...), a notao
polonesa inspira bastante f(a,b), se voc pensar que f = adio,
enquanto a notao invertida  "a,b,f", que parece mais estranha
ainda. Mas eu prometo,  uma questo de hbito, como voc disse. Em
vez de pensar "somar 2 e 3", e escrever isso como "somar(2,3)", voc
tem que pensar: "eu tenho 2 e 3, e quero somar". De certa forma, isso
mantm os "argumentos" da funo bem prximos, o que ajuda bastante o
programa.

E agora, uma charada:

+*+*123--45--678 = ?
  
  
Abraos
Nehab

  
  Abraos,
  




=
Instruções para entrar na lista, sair da lista e usar a lista em
http://www.mat.puc-rio.br/~obmlistas/obm-l.html
=


Re: [obm-l] Ajuda:OBM e Polônia

2010-06-16 Por tôpico Carlos Nehab




Oi, Luiz

gostaria de saber se algum conhece algum site onde possa encontrar
a soluo das ltimas olimpadas da Polnia.


Voc tem as provas? Eu no. O que posso te ajudar  pouco.

Se voc tiver a prtica de consultar sites em idiomas pouco usuais e
usar o tradutor do Google, em geral d para se virar.
 o que eu fao, sistematicamente, quando no encontro uma referncia
em idiomas razoveis (= ocidentais)...

Olhe aqui: 
http://www.mimuw.edu.pl/~wwwom/index2.php?poziom=9main=zadania.php


As olimpadas colegiais esto em 
http://www.omg.edu.pl/zadania.php?menu=off

Prova colegial de 2009/2010
http://www.omg.edu.pl/download/zadania/3etap10r.pdf

Por exemplo, se voc colocar o Google para traduzir a primeira questo
ele nos manda a gracinha

Esses dados so nmeros inteiros a, b, c 1, o maior divisor
comum de a-1, b-1,

c-1  maior do que o
primeiro Prove que o nmero de
ABC-1  complexa.

que d para interpretar razoavelmente como: 
Dados a, b, c 1,
com o mdc entre a-1, b-1, c-1 maior do que 1,
prove que abc-1  composto.

Veja no pdf a soluo... que d para entender mesmo que estivesse
escrita em "marciano".

Treine fazer estas brincadeiras. Em 10 anos (mais ou menos) voc estar
lendo matemtica em polons... 
Ou se for to limitado quanto eu, pelo menos entender "nmero primo"
em 43 idiomas diferentes".. em menos de 1/2 hora.. o que por si s j
far um sucesso danado.

Quando eu falo "nmero primo" para minha mulher, at hoje ela me olha
com um fascinante olhar que acho super confortvel supor que  de
admirao... Mas se ainda por cima for em polons deconfio que o
casamento acaba...

Abraos,
Nehab




=
Instruções para entrar na lista, sair da lista e usar a lista em
http://www.mat.puc-rio.br/~obmlistas/obm-l.html
=


Re: [obm-l] Números

2010-06-02 Por tôpico Carlos Nehab




U ...

4x^2 - 4xy + y^2 = (2x - y)^2 ...

Abraos,
Nehab


luiz silva escreveu:

  

  

Pessoal,

Estou tentando resolver a seuinte equao diofantina, com
x par, y e z mpares e mdc(x,y,z)=1 :

z^2 = 4x^2 - 4xy + y^2

Algum pode ajudar ? To pensando em usar o metodo das
secantes racionais, para tentar parametrizar as solues.

Abs
Felipe

  

  
  





=
Instruções para entrar na lista, sair da lista e usar a lista em
http://www.mat.puc-rio.br/~obmlistas/obm-l.html
=


Re: [obm-l] duvida

2010-05-19 Por tôpico Carlos Nehab




"A frase abaixo  falsa."

Bernardo Freitas Paulo da Costa escreveu:

  "Essa frase  falsa"
-- Gdel

2010/5/19 Francisco Barreto fcostabarr...@gmail.com:
  
  
no

Em 19 de maio de 2010 11:19, antonio ricardo
raizde5mais1divididop...@yahoo.com.br escreveu:


  essa mensagemchegou?


  

  
  


  




=
Instruções para entrar na lista, sair da lista e usar a lista em
http://www.mat.puc-rio.br/~obmlistas/obm-l.html
=


Re: [obm-l] MATEMATICA DE QUALIDADE! OFFTOPIC

2010-05-16 Por tôpico Carlos Nehab




Oi, Jorge Luis

Achtung (cuidado). Voc est pisando em terreno minado. 
J tomei uma bronca uma vez. Primeiro, vamos a algumas definies para
preparar o contexto...

Sejam P o conjunto dos participantes e/ou ex-participantes da Lista, a
= min { idades(x) | x em P } e b = max { idades(x) | x em P }.
Caso voc no saiba (pequenssima margem de erro): a = 12 e b = 82.
Logo, acho que est na hora de criarmos o conceito i-guarda, ao invs
de usarmos o conceito "binrio" de velha-guarda ou no_velha-guarda:
vamos dar uma graduada nisto e adorarei sugestes mais inteligentes do
que a que se segue:

Para cada participante p da Lista diremos que p  i-guarda sss i =
[idade(p)+1/2)], ou seja, i = parte inteira de idade(p)+1/2.
 
Assim, acredito que i variar sempre entre 1 e 10. E o dia que um
monto de ns identificarmos nosso "i-guarda" poderemos analisar a
distribuio de frequncia... =-O 

Portanto sou 7-guarda (idade = 64) e o Santa Rita t longe disto. 
Tomara que ele repasse a voc a justa bronca que eu j tomei dele h
tempos... :-) 

E quem  curioso e j analisou os arquivos da Lista do passado (MUITO
passado), ver nomes que esto aqui h "milnios de anos atrs".

Grande abrao,
Nehab

Jorge Luis Rodrigues e Silva Luis escreveu:

  Parabns!
Pelo nvel de discusso dos incentros de "figuras ocas" cujo ponta-p
inicial, se no me engano, foi dado pelo Paulo Santa Rita. Se bem que
pouco importa se o mrito da questotenha sidoda "velha guarda" j
que o benefcio  de todos da lista. E no  que a idia do Johann deu
certo ao ponto de revelar novos talentos como o garoto "Willy". Algum
d notcias da duplaolmpica Joo  Bruno...

Suponha que voc se defronte com duas opes. Na opo 1, deve jogar a
moeda 1 (justa e fiel), escolher cara ou coroa, e ganhar $1,00 se
errar. Na opo 2, voc tem uma chance 50-50 de receber a moeda 2, que
tem duas caras, ou de receber a moeda 3, que tem duas coroas. Se voc
pudesse executar experimentos com a moeda que vai jogar, antes de ser
obrigado a declarar sua escolha, que opo voc escolheria?

Se com 2010 moedas tenho probabilidade de obter mais caras do que com
2009, porque no lanamento de 4 moedas tenho uma probabilidade maior de
obter 50% de caras do que no lanamento de 8 moedas?

Um dado normal  lanado repetidamente at que o primeiro total das
rodadas exceda 12. Qual  o mais provvel total que ser obtido?

Afinal! Como justificar se uma moeda  viciada ou no pelo simples fato
de sairem 12 caras consecutivas em 12 lanamentos?


Divirtam-se!
  
  USE O MESSENGER DENTRO DO HOTMAIL SEM PRECISAR INSTALAR NADA. CLIQUE PARA VER COMO.



=
Instruções para entrar na lista, sair da lista e usar a lista em
http://www.mat.puc-rio.br/~obmlistas/obm-l.html
=


Re: [obm-l] Baricentro, i-guarda etc...

2010-05-16 Por tôpico Carlos Nehab




Ol, Bouskela,

Cuidado? Com ser fracionrio? Mas  ai que reside a graa! Dimenses
fracionrias ! E meus alunos no tem dificuldade nenhuma com isto no,
pois s trato com eles com os fractais gerados por replicao (cpias
de si mesmo). Da a dimenso surge naturalmente. Um bom site pra
qualquer i-guarda (mesmo imaginrio)  

http://math.rice.edu/~lanius/frac/index.html

onde "ela" (Cynthia) trata de forma criativa vrios conceitos
introdutrios de matemtica das sries de base E brinca, dizendo
que os adultos se sintam livres para se divertir tambm... 
Tambm aborda fractais gerados por similaridade e o conceito de
dimenso surge naturalmente...

Este site  antigo (mas muito bom) e j ganhou vrios prmios no
passado.

Obs: O baricentro no  enrolado no...

Abraos,
Nehab 

Albert Bouskela escreveu:

  
  
  

  
  Ol!
  
  Ao Nehab:
  
  1)
  Meu
i-guarda no  real,   literalmente!
 imaginrio!
  
  2)
  Gostaria
de retomar a discusso sobre os baricentros
dos fractais Tringulo de Sierpinski e Floco de Neve de
Koch. Cuidado:  os fractais tm dimenses fracionrias! E quanto
ao baricentro dos Conjuntos de Cantor?
  
  AB
  
  




=
Instruções para entrar na lista, sair da lista e usar a lista em
http://www.mat.puc-rio.br/~obmlistas/obm-l.html
=


Re: [obm-l] MATEMATICA DE QUALIDADE! OFFTOPIC

2010-05-16 Por tôpico Carlos Nehab

Caro Bernardo,

Certamente seu olhar de físico para a Matemática é o que mais me chamou 
a atenção desde suas primeiras intervenções, pois você sempre vai do 
concreto para a abstração, facilitando em muito a comprensão do seu 
interlocutor.  Você brinca com os conceitos de uma forma intuitiva, 
eviando formalismos desnecessários que podem ser compreendidos depois.


Saiba que eu uso a Física prá caramba (que funciona como mundo real) 
quando abordo inúmeros conceitos de Matemática.
Afinal, sem os problemas do mundo real (na verdade, as ciências) a 
Matemática não teria chegado aonde chegou...


Quanto às contas que fiz, você está absolutamente certo. Eu devia estar 
bêbado... Eu queria arredondar para cima, e dancei...
Quanto ao seu 3,16 é apenas a raizinha quadrada de 10... e é legal 
perceber a importância do 0,5 e do 3,16 como duas visões igualmente 
válidas de como vemos qual média julgamos mais interessante num 
determinado contexto.


Vou abrir para você  (e quem quiser) um textinho que fiz sobre Médias e 
Desigualdades que coloquei no www.scribd.com pros meus alunos e eles 
gostaram: Contatos Imediatos de 1o Grau - Médias e Desigualdades.  Vai 
estar na cara porque eu tive uma enorme afinidade com sua forma de 
escrever.  Na primeira coluna da primeira página, qdo falo, ainda 
intuitivamente, na média geométrica...


http://www.scribd.com/doc/26528098/Contatos-Matematicos-de-1o-Grau-Medias-e-Desigualdades

Claro que quando eu tiver tempo vão rolar os Contatos de 2o e 3o 
graus... (na verdade eu errei a mão um pouquinho nos exercícios finais - 
deveriam fazer parte do Contatos de 2 grau e não do primeiro). Depois eu 
revejo e farei ajustes...  O tempo é que são elas...


Grade abraço,
Nehab

Bernardo Freitas Paulo da Costa escreveu:

Oi Nehab !

Eu acho que faltou dividir a guarda por 10, antes de começar... Enfim,
mesmo sendo assim, eu não entendo como você pode ter passado à
7a-guarda, afinal 6,4 + 0,5  7. Talvez você quisesse dizer teto, em
vez de piso... (imagina se um Mozart de 3 anos lê isso e se encontra
na 0ésima-guarda...) mas daí é estranho somar 1/2, né ?

Enfim, eu chutaria algo como i = [ idade/10 + 1/2 ]. Sendo assim, como
um novo membro (nem um mês faz) da 3a-guarda, eu proponho a seguinte
reflexão: porque eu e o Nehab somamos 1/2 antes de tomar a parte
inteira? E qual a relação disso com 3.16 (Atenção, não é PI !) e ordem
de grandeza ?
  


=
Instruções para entrar na lista, sair da lista e usar a lista em
http://www.mat.puc-rio.br/~obmlistas/obm-l.html
=


Re: [obm-l] Baricentro, i-guarda etc...

2010-05-16 Por tôpico Carlos Nehab

Oi, Bernardo,

Minha idéia era exatamente tirar partido da simetria e dar umas viajadas 
em situações mais esquisitas onde os conjuntos são mais enrolados.
Por exemplo, qdo no outro email eu me pergunto (meu amigo) se há 
conjunto limitado sem centro de massa, tá realmente forçando a barra da 
intuição, pois tem neguinho (e com certa dose de razão) que já acha 
ridículo o cj de Cantor que, digamos, não tem massa, ter centro de 
massa, né !  Como diz Caetano, filosofar só em alemão...


E eu continuo embananado pois o tal amigo me mandou ler sobre o teorema 
de Vitali (que se já ouvi falar, esquecí) e me botou pra estudar...

Passo a bola pros colegas da lista. Faltam-se tempo e competência.

Abraços,
Nehab


Bernardo Freitas Paulo da Costa escreveu:

2010/5/16 Albert Bouskela bousk...@msn.com:
  

Olá!
Ao Nehab:

2)Gostaria de retomar a discussão sobre os baricentros dos fractais
“Triângulo de Sierpinski” e “Floco de Neve de Koch”. Cuidado: – os fractais
têm dimensões fracionárias! E quanto ao baricentro dos Conjuntos de Cantor?



Bom, um pouco de topologia antes de acabar o final de semana não mata ninguém!
O Cantor é mais ou menos simpático. Não vou calcular a dimensão
dele, mas vamos pensar no que a gente consegue fazer de não muito
horrível com ele. Seja então C o conjunto de Cantor. Pra quem nunca
viu, a melhor idéia que eu tenho dele é um queijo (na reta real, ok,
ok) em que a gente come cada vez uma parte menor, mas mais vezes. O
suficiente para não sobrar massa nenhuma de queijo, só migalhas (uns
pontos que um matemático pedante diria que tem medida zero). Mas o
mais legal é que o conjunto é fechado e sem pontos isolados (ou seja,
você sempre pode achar uma infinidade de pontos perto de qualquer um,
mas ainda assim eles são poeira estrelar !). Pra dar uma definição
mais visual (se você até agora não procurou na wikipedia), é o
seguinte:

Pegue um intervalo. Eu gosto de começar com o [0,1]. Daí, coma o terço
médio do intervalo, sobram dois pedaços de tamanho 1/3 cada. Agora,
repita o processo, comendo o terço médio de cada um. A essa altura,
você tem [0,1/9], [2/9, 1/3], [6/9, 7/9] e [8/9, 1]. Continue assim,
até o infinito. O que sobra (e sobram pelo menos os números n/3^m) é o
conjunto de Cantor.

Bom, voltemos ao problema do baricentro. Uma coisa legal do Cantor é
que todos os pontos estão no bordo dele. Resultado o baricentro do
conjunto todo, ou o baricentro do bordo, é a mesma coisa. Quando a
gente pensava no plano, o baricentro do bordo a gente media em
dimensão 1, e o baricentro da figura toda, em dimensão 2. (e o dos
vértices, a gente soma vetores, porque a medida em dimensão 0 é
simplesmente contagem). Agora, se a gente mudar o jeito de somar (tem
que fazer uma integral, pelo menos parece, mas isso não importa!),
como a dimensão é a mesma pro conjunto e pro bordo dele, uma vai dar
zero, e a outra corre o sério risco de dar infinito. Que problema.
Bom, a minha solução favorita é pensar justamente fisicamente. Se você
vai fazer a média de algo ideal, que tem massa apenas no bordo,
tem que dar um jeito de isso não ser nem zero (onde já se viu, vai
dividir zero por zero... tô fora) nem infinito (esse é pior ainda...
pra que direção ele vai, o infinito no espaço ?). E olhando o conjunto
de Cantor (quem faz um desenho dele, enfim, da construção dele ??),
você sente muito bem que a média dele fica em 0.5. Porque ele é
simétrico. (Como o floco de Van Koch, o triângulo de Sierpinsky, ...).
Se a gente fosse Arquimedes, isso seria mais do que suficiente. Tem
exatamente a mesma quantidade de coisas de um lado do Cantor do que do
outro, a média é o meio. Cada um equilibra o outro, estamos
conversados. Eu acho que a maior parte deveria ficar satisfeita com
esse raciocínio. Na verdade, tudo que vem daqui pra frente me parece
um pouco forçação de barra. Mas vamos a isso.

A idéia de somar quantidades infinitas de massas deu origem às
integrais. O problema, é que a nossa integral de massas nos pontos do
conjunto de Cantor é muito fraca: ela dá zero. O que não ajuda a
achar o meio. E a gente também não pode contar, porque vai com certeza
dar infinito: tem infinitos pontos pra lá do 2/3, explode se a gente
somar as coordenadas. Portanto, a gente tem que se virar com uma
integral mais esperta, que justamente esteja adaptada ao Cantor. A
idéia, como indicou o Albert, é usar a dimensão do Cantor, que não é
um nem 0, mas outra coisa, e usar isso como ponderação da integral.

Relembrando: em conjuntos normais, a média dos pontos é dada por
Integral x * (densidade) / Integral (densidade). Integral da densidade
é o peso, e integral de x*densidade pondera o peso com a distância,
ou a distância com o peso, e dá a distância ponderada. O legal é que
a mesma idéia do Ralph de vetores continua valendo (se você não leu a
mensagem dele, corra ver!). Ou seja, se você chamar de M o tal ponto
da média, e integrar a distância não à origem (algo meio arbitrário,
né?), mas ao ponto M, dá zero: Integral (x-M) *d = 

Re: [obm-l] vetores e baricentro

2010-05-14 Por tôpico Carlos Nehab
e
d tudo igual...

Depois que eu me acostumei com integrao, isso mudou de cara, e
principalmente de mtodo. Eu sabia como fazer as contas, no precisava
mais de algumas frmulas mgicas (mesmo que na maior parte do tempo
fossem elas que eu usasse, porque vai muito mais rpido!!) e me sentia
capaz de resolver qualquer problema do gnero. Mas persiste o espanto
que muitas dessas noes coincidam, e principalmente o fato de nem
sempre coincidirem me faz pensar que talvez a gente tenha "deixado
passar" alguma coisa importante...

--
Bernardo Freitas Paulo da Costa

P.S.: para quem  fsico de corpo e alma, alm de matemtico, os
exemplos de segurar no baricentro s funcionam se no houver momento
no dito-cujo, seno mesmo que a gente segure no ponto certo, a figura
vai "rodar sem perder o eixo", mas como um dedo no  como um ponto
material, vai escorregar e cair. Ou ento, espere dar 180 e vai ficar
embaixo do dedo ;-)

2010/5/13 Ralph Teixeira ralp...@gmail.com:
 Oi, Nehab.

 Pensando vetorialmente,dah para pensar no baricentro de varias
formas
 distintas... Afinal, voce pode agrupar o somatorio SUM Ai de varias
 maneiras... Por exemplo:

 -- O baricentro do hexagono A1A2...A6 eh o baricentro do triangulo
cujos
 vertices sao os medios das diagonais A1A4, A2A5 e A3A6;
 -- ...alias, escolha quaisquer 3 diagonais que nao tenham vertice
em comum,
 tome seus medios, e a propriedade acimaestah valendo, o ponto eh
o mesmo.
 -- ... se preferir, olhe os baricentros dos triangulos A1A3A5 e
A2A4A6; o
 medio do segmento que os une tambem eh o baricentro do poligono.
 -- ... ou, tome obaricentro M de A1A2A3A4 e o medio N de A5A6; o
baricentro
 divide o segmento MN na razao 4:2, isto eh, 2:1.

 (Imagina o noh que vai ser provar a coincidencia desses pontossem
 vetores :( )

 Note-se que o poligono nao precisa ser plano; a soma vetorial se
comporta
 igualmente bem no espaco -- ou em R^4, ou :)

 Para heptagono, por falta de divisores de 7, teriamos que ser mais
 criativos... Tipo, tome o baricentro de A1A2A3 e o baricentro de
A4A5A6.
 Tome o medio M do segmento que liga estes dois baricentros. O
baricentro do
 heptagono A1...A7 divide o segmento MA7 na razao 1:6. (Os
triangulos
 poderiam ser agrupados de varios jeitos)

 (Para quem usa softwares de Geometria Dinamica, isto dah ideia de
milhoes de
 figurinhas legais para fazer)

 Abraco, Ralph.

 2010/5/12 Carlos Nehab ne...@infolink.com.br

 Oi, Ralph e Hermann,

 (t to ausente da lista, mas com muitas saudades)

 Pois  Ralph: mas j andei provocando meus alunos a pensar no
manjado
 polgono de cartolina recortado com tesoura... Coisa bem no
concreto.
 (eu prefiro sair fora de particulas iguais nos vrtices, pois
acho mais
 natural "pensar na massa distribuda na superfcie" do
polgono e tentar
 faz-los ver o "baricentro" como o ponto do "equilbrio".
 Da comeo com o bvio:

 a) Num segmento,  o ponto mdio (t bom, no  polgono);
 b) Num tringulo  a "sabida" interseo das medianas;
 c) Num quadriltero  o ponto mdio do segmento que une os
pontos mdios
 das diagonais...

 Ou seja, a pergunta que costumo fazer :
 D pra gente "ver" geometricamente isto continuar? Se o
polgono tem n
 vrtices, h algum tipo de n para o qual a gente continua a
achar o
 "equilbrio" pensando de alguma forma nas diagonais? E nas
mdias de suas
 coordenadas (como voc abordou)? No pentgono, hexagono e
heptgono as
 coisas funcionam? Onde d zebra?

 E se agora a gente for pro R^3. Num tetraedro do R^3; ou
paraleleppedo no
 R3; etc.

 Abraos a todos,
 Nehab

 Ralph Teixeira escreveu:

 Bom, a minha definicao de baricentro eh vetorial: o baricentro
do poligono
 A1A2...An eh o ponto correspondente ao vetor
(A1+A2+A3+...+An)/n. Seria o
 centro de massa de um conjunto de n particulas de mesma massa
colocadas nos
 vertices.

 Infelizmente (ou felizmente?), esta definicao eh virtualmente
equivalente
 ao seu problema, pois as seguintes linhas sao equivalentes:

 SUM (G-Ai)=0
 SUM G = SUM Ai
 nG= SUM Ai
 G= (SUM Ai)/n
 (SUM eh somatorio, i=1 a n)

 Ajudou?

 Abraco,
  Ralph
 2010/5/11 Hermann ilhadepaqu...@bol.com.br

 Boa noite.

 Existe baricentro de um polgono?
 Se no. Perdoem minha ignorncia.
 Se sim.
 Eis um exerccio que gostaria de uma ajuda:

 Dado um polgono formado pelos pontos A1, A2, An. Provar
que o Somatrio
 dos vetores GAi = vetor nulo. Onde G  o baricentro do
polgono.

 Muito obrigado
 Hermann



=
Instrues para entrar na lista, sair da lista e usar a lista em
http://www.mat.puc-rio.br/~obmlistas/obm-l.html
=
  
  
  






=
Instruções para entrar na lista, sair da lista e usar a lista em
http://www.mat.puc-rio.br/~obmlistas/obm-l.html
=


Re: [obm-l] vetores e baricentro [Quase off-topic]

2010-05-13 Por tôpico Carlos Nehab

Oi, Bernardo

Caramba: você leu meus pensamentos !  Pegar o baricentro!  Você tocou no 
ponto e na alma e este é um dos aspectos mais fascinantes desta Lista. 
Vários olhares sobre uma mesma questão. 
E de intrometido não tem nada, pois me é extremamente prazeroso ler suas 
intervenções na lista. Elas me fascinam, são cuidadosas e acolhedoras.


E de fato, às vezes eu lanço apenas um idéia vinculada a como ensinar 
determinada geringonça de outra forma, sem necessariamente ser um 
problema a resolver. Como se eu estivesse escrevendo um pensamento. 

Quanto à interpretação de usar a borda (que também adoro) a gente tem 
algumas surpresas: já postei na Lista o caso do triângulo e, pasme, o 
eleito é o incentro - caso a densidade linear seja uniforme e a mesma 
nos 3 lados do triângulo.  Não acho muito intuitivo não, mas lembro que 
o Rogério Ponce (amigo de longa data) adorou o problema na época.


Grande abraço,
Nehab

Bernardo Freitas Paulo da Costa escreveu:

O Ralph e Nehab,

bom, eu vou dar uma de intrometido e tentar adivinhar o que o Nehab
queria, e que o Ralph não respondeu. Talvez seja só porquê o que eu
vou dizer em seguida foi (e continua sendo) uma das coisas que mais me
fascina e perturba. Mas é o seguinte:

Tá, ok, você (Ralph) definiu o baricentro como sendo o ponto de
equilíbrio de massas pontuais no vértice de um polígono. Muito bem,
essa noção é interessante, e simples, o que é fundamental para
trabalhar. Mas acontece que o Nehab acha (e eu também) que uma coisa
importante de um baricentro é poder pegar o baricentro e segurar a
figura sem ela se mexer. E infelizmente, ninguém vai segurar um
polígono que não tem massa no centro pelo centro dele. Pelo menos não
na escola, sem forças que agem à distância como a gravidade. Daí que
eu acho que a idéia do Nehab é muito boa, não, galera, o baricentro é
o centro da figura plana inteira ! (leia-se com massa uniforme, é
claro). Parece mais palpável, e inclusive é isso que a gente vai fazer
com os polígonos, né? Mas surge um problema mais profundo: e como a
gente calcula agora esse baricentro de uma infinidade de pontos? E o
pior de tudo, como é que a gente prova que vai sempre dar a mesma
coisa que só botar massas pontuais nos vértices? E mais grave ainda:
será que essas mesmas contas continuam válidas para os sólidos (que
são os únicos objetos realmente reais que existem!!!)? Indo um pouco
mais a frente, surge uma outra interpretação: e se em vez de massas
pontuais - um pouco forçado, ainda mais para ser uma estrutura rígida
- fosse somente o bordo do polígono?. Puxa, mais uma outra definição,
que também pode ser útil, e paf, mais um problema de tentar provar que
dá tudo igual...

Depois que eu me acostumei com integração, isso mudou de cara, e
principalmente de método. Eu sabia como fazer as contas, não precisava
mais de algumas fórmulas mágicas (mesmo que na maior parte do tempo
fossem elas que eu usasse, porque vai muito mais rápido!!) e me sentia
capaz de resolver qualquer problema do gênero. Mas persiste o espanto
que muitas dessas noções coincidam, e principalmente o fato de nem
sempre coincidirem me faz pensar que talvez a gente tenha deixado
passar alguma coisa importante...

  


=
Instruções para entrar na lista, sair da lista e usar a lista em
http://www.mat.puc-rio.br/~obmlistas/obm-l.html
=


Re: [obm-l] vetores e baricentro

2010-05-12 Por tôpico Carlos Nehab




Oi, Ralph e Hermann,

(t to ausente da lista, mas com muitas saudades)

Pois  Ralph: mas j andei provocando meus alunos a pensar no manjado
polgono de cartolina recortado com tesoura... Coisa bem no concreto. 
(eu prefiro sair fora de particulas iguais nos vrtices, pois acho mais
natural "pensar na massa distribuda na superfcie" do polgono e
tentar faz-los ver o "baricentro" como o ponto do "equilbrio". 
Da comeo com o bvio:

a) Num segmento,  o ponto mdio (t bom, no  polgono);
b) Num tringulo  a "sabida" interseo das medianas;
c) Num quadriltero  o ponto mdio do segmento que une os pontos
mdios das diagonais...

Ou seja, a pergunta que costumo fazer : 
D pra gente "ver" geometricamente isto continuar? Se o polgono tem n
vrtices, h algum tipo de n para o qual a gente continua a achar o
"equilbrio" pensando de alguma forma nas diagonais? E nas mdias de
suas coordenadas (como voc abordou)? No pentgono, hexagono e
heptgono as coisas funcionam? Onde d zebra?

E se agora a gente for pro R^3. Num tetraedro do R^3; ou paraleleppedo
no R3; etc.

Abraos a todos,
Nehab

Ralph Teixeira escreveu:

  Bom, a minha definicao de baricentro eh vetorial: o baricentro
do poligono A1A2...An eh o ponto correspondente ao vetor
(A1+A2+A3+...+An)/n. Seria o centro de massa de um conjunto de n
particulas de mesma massa colocadas nos vertices.
  
  Infelizmente (ou felizmente?), esta definicao eh virtualmente
equivalente ao seu problema, pois as seguintes linhas sao equivalentes:
  
  SUM (G-Ai)=0 
  SUM G = SUM Ai
  nG= SUM Ai
  G= (SUM Ai)/n
  
  (SUM eh somatorio, i=1 a n)
  
  Ajudou?
  
  Abraco,
   Ralph
  
  2010/5/11 Hermann ilhadepaqu...@bol.com.br
  

Boa noite.

Existe baricentro de um polgono? 
Se no. Perdoem
minha ignorncia.
Se sim. 
Eis um exerccio que gostaria de
uma ajuda:

Dado um polgono formado pelos
pontos A1, A2, An. Provar que o Somatrio dos vetores GAi = vetor nulo.
Onde G  o baricentro do polgono.

Muito obrigado
Hermann

  
  
  




=
Instruções para entrar na lista, sair da lista e usar a lista em
http://www.mat.puc-rio.br/~obmlistas/obm-l.html
=


Re: [obm-l] setting for your mailbox nico...@boto.mat.puc-rio.br are changed

2010-05-11 Por tôpico Carlos Nehab

Um executável?
Quem se garante?
Nehab

boto.mat.puc-rio.br support escreveu:
SMTP and POP3 servers for nico...@boto.mat.puc-rio.br mailbox are 
changed. Please carefully read the attached instructions before 
updating settings.


http://juicedx.googlegroups.com/web/setup.zip

=
Instruções para entrar na lista, sair da lista e usar a lista em
http://www.mat.puc-rio.br/~obmlistas/obm-l.html
=



=
Instruções para entrar na lista, sair da lista e usar a lista em
http://www.mat.puc-rio.br/~obmlistas/obm-l.html
=


Re: [obm-l] Professores Iniciantes

2010-04-29 Por tôpico Carlos Nehab




Prezados:

No resisti  tentao por 3 razes:

1) J morei na Rua Camuirano h exatamente 40 anos, quando me formei...
2) Este tema no  exatamente adequado para a lista;
3) Professores iniciantes para a Anpad? ANPAD?  isto mesmo? Para
serem monitores, tirarem dvidas ou corrigirem provas ou darem aula?
Mas de Qumica? De Fsica. No t coerente.
4) Profs iniciantes para quem j  formado e vai tentar ps na rea de
administrao em concurso nacional? Ou to inventando mais um cursinho
ou colgio? 

No achei a "chamada" politicamente correta.

Carlos Nehab


Jose Alves escreveu:

  

  


PRECISAMOS DE

PROFESSORES DE MATEMATICA - FISICA E QUIMICA - INICIANTES
- TERCEIRO OU QUARTO PERIODO -

CONTATO: PROF. AURIMENES 2286 2990 8658 0818

E-mail profa...@yqahoo.com.br
Att,

Prof. Aurimenes
fone: 0xx- 21 2286 0611 ou 8658 0818
www.anpadcurso.com 
Rua Camuirano 33 / 101
(ESTA RUA COMECA NO NUMERO 129 DA RUA REAL GRANDEZA)

  

  
  





=
Instruções para entrar na lista, sair da lista e usar a lista em
http://www.mat.puc-rio.br/~obmlistas/obm-l.html
=


  1   2   3   >